PA 532 ER/Surgery Questions

Réussis tes devoirs et examens dès maintenant avec Quizwiz!

A 48-year-old male with a past medical history of thyrotoxicosis and medication non-adherence is seen at an outpatient clinic. He has not been taking the prescribed methimazole for the last three weeks. He complains of palpitations at rest, frequent loose bowel movements, nausea, hand tremors, and diaphoresis. On examination his heart rate is 122/min, respiratory rate is 26/min, the temperature is 103 degrees F, and BP 142/88. He does not have proptosis or conjunctival erythema. He seems confused on multiple occasions during the encounter. What do you expect to find in his lab works? A. Normal TSH, high FT4, hypercalcemia, low PTH, transaminitis B. Low TSH, high FT4, hypercalcemia, high PTH, transaminitis C. Low TSH, high FT4, hypocalcemia, high PTH, normal LFTs D. High TSH, high FT4, hypocalcemia, Low PTH, normal LFTs

A. Normal TSH, high FT4, hypercalcemia, low PTH, transaminitis

Which of the following pathophysiological processes is believed to initiate acute appendicitis? A. Obstruction B. Perforation C. Hemorrhage D. Vascular compromise

A. Obstruction: Fecalith (Stone made of feces) → Bacteria overgrowth → Rupture if untreated

A 22 year-old male presents to the clinic complaining of scrotal pain that radiates into the groin. Patient admits to being a weightlifter and was lifting 24 hours prior to this pain developing into the scrotum. The patient admits to being sexually active with only his male partner. Examination reveals a reddened scrotum and it is difficult to distinguish the epididymis from the testes on the right side. Elevation of the right testicle brings relief of the pain. This is known as a positive A. Prehn's sign. B. Cullen's sign C. Rovsing's sign D. Murphy's sign

A. Prehn's sign Cullen's sign: Bruising around umbilical Rovsing's signL LLQ touch → RLQ pain Murphy's sign: Cholecystitis

A 17-year-old male is brought to the emergency department after getting shot in the eye by a BB gun. On initial assessment, the patient has a subconjunctival hemorrhage of the left eye with deformity of the globe. The patient's vision in that eye is 20/400 at three feet. The clinician uses a fluorescein strip and finds the patient to be Seidel positive at the limbus, extending back into the sclera at the two o'clock position. The clinician decides that the patient needs urgent surgical intervention. Given this patient's clinical presentation, what anesthetic would be preferred in this patient? A. Propofol B. Nitrous oxide C. Succinylcholine D. Ketamine

A. Propofol --> Globe rupture Dec ICP Agents that ICP are CI --> Nitrous oxide, succinylcholine, ketamine

A 70-year-old woman with history of Factor V Leiden presents to the ER with tachypnea, chest pain, and a new cough. Further questioning reveals she just got back from a trip to Europe, which she took a 10-hour flight for. After a few hours, her blood pressure starts to drop dramatically and she starts experiencing visual issues. What is the underlying condition that is causing her shock? A. Pulmonary embolism B. Sepsis C. Hemorrhagic D. Anaphylaxis

A. Pulmonary embolism

Kussmaul breathing is characterized by A. Rapid, deep labored breathing B. Irregular and varying depth of breathing C. Frequently interspersed deeper breaths D. Periods of deep breathing alternate with periods of apnea When might you see it?

A. Rapid, deep labored breathing DKA, metabolic acidosis Periods of deep breathing alternate with periods of apnea= Chyenne stokes breathing, result of heart failure, inc risk of sudden cardiac death and worse outcome

A 16-year-old female presents to ED after being accidentally hit in the head by a team member's elbow while playing basketball just prior to arrival. Which of the following symptoms would be suggestive of development of a concussion? A. Palpitations B. Dizziness C. Tinnitus D. Weakness of the legs

B. Dizziness

Create a multiple-choice question representing the diagnostic findings of choledocholithiasis. All of the choices must be relevant, but there must be a clear best answer. Which of the following diagnostic findings is the gold standard for identifying choledocholithiasis? A. Ultrasound B. ERCP C. Elevated serum bilirubin D. Abdominal CT

B. ERCP

Which of the following is a diagnostic finding of Tylenol toxicity? A. Respiratory alkalosis B. Elevated serum transmaninase C. Metabolic acidosis D. Elevated BNP

B. Elevated serum transmaninase

A 36-year-old male developed a sore throat and was treated with IM penicillin. Within 20 minutes, he felt faint and became dyspneic. Upon entry to the emergency department, he was pale and apprehensive. He had a thready pulse, and systolic blood pressure was 40 mmHg. Which of the following is the most appropriate initial agent to use? A. Dopamine B. Epinephrine C. Hydrocortisone D. Diphenhydramine

B. Epinephrine

5 year old patient comes into the ED complaining of RUQ pain. He hasn't been eating and has been vomiting for the past couple days. His parents says that he had a recent fever and treated it with an OTC medication but can't remember what it was. Labs show elevated serum transaminase and hypokalemia. If you were to take an ABG, what would it show? A. Metabolic acidosis B. Metabolic alkalosis C. Respiratory acidosis D. Respiratory alkalosis

A. Metabolic acidosis

Which of the following is a diagnostic finding of a heroin overdose? A. Miosis B. Mydraisis C. Tachycardia D. Profuse secretions

A. Miosis

A 16-year-old female presents to the emergency department with complaints of nausea, vomiting, and abdominal pain after she ingested 30 pills of an unknown drug 6 hours ago. Her family does not know what kind of pills she took. She was given intravenous sodium bicarbonate, but her symptoms did not resolve. Which of the following is responsible for her condition? A. N-acetyl benzoquinone imine (NAPQI) B. Aspirin (acetylsalicylic acid) C Cocaine D. Morphine

A. N-acetyl benzoquinone imine (NAPQI) --> Acetaminophen toxicity Morphine, aspirin and cocaine would all be alkalized by sodium bicarb

The first step in the treatment of a patient with an intestinal obstruction and no comorbid diseases is A. Nasogastric decompression B. Invasive hemodynamic monitoring C. Abdominal exploration D. Administration of antibiotics

A. Nasogastric decompression

Which of the following statements regarding the management of mild traumatic brain injury is INCORRECT? A. No cognitive impairment is expected in mild traumatic brain injury. B. Ensure adequate follow-up of patients. C. Athletes should have a graded return to play. D. Management includes symptom control. E. It is important to prevent re-injury.

A. No cognitive impairment is expected in mild traumatic brain injury.

60-year-old Asian American woman presents with sudden ocular pain. She reports she was visiting the planetarium when the pain started and when she walked outside, she saw halos around the streetlights. The pain was so bad that she began to vomit. She reports her vision is decreased. Physical examination reveals conjunctival injection, a cloudy cornea, and pupils. What is the diagnosis and pathophysiology of this condition

Acute closed angle glaucoma Inc IOP --> Blockage of drainage of aqueous humor

WBC count of 50k

Acute leukemic blast crisis

What is Kehr's sign? Explain the mechanism

Acute pain in the tip of the shoulder due to the presence of blood or other irritants in the peritoneal cavity when a person is lying down and the legs are elevated Ruptured spleen

In a patient with a pneumothorax following a stab wound, the chest tube is best inserted at which level? A. Between the second and third intercostal spaces B. Between the eighth and ninth intercostal spaces C. Between the fourth and fifth intercostal spaces, just anterior to the mid-axillary line D. Just below the clavicle

C. Between the fourth and fifth intercostal spaces, just anterior to the mid-axillary line Too high --> Subclavian vessel damage Too low --> Liver or spleen damage Avoid tracers PG pt --> Place level or two high due to higher diaphragm

Which of the following is most helpful in the diagnosis of a retropharyngeal abscess? A. CBC with differential B. Fever and muffled voice on examination C. CT of neck with contrast D. Hx of recent throat infection

C. CT of neck with contrast

A 45 YOM presents with pain to his RUQ. He has some N/V associated with his pain. Upon abdominal US and CT there is dilation of he bile duct. What does he have? A. Cholangitis B. Cholelithiasis C. Choledocholithiasis D. Cholecystitis

C. Choledocholithiasis

An 18-year-old patient has a tibia/fibula fracture following a motorcycle crash. Twelve hours later the patient presents with increased pain despite adequate doses of analgesics and immobilization. Which of the following is the most likely diagnosis? A. Avascular necrosis B. Myositis ossificans C. Compartment syndrome D. Reflex sympathetic dystrophy

C. Compartment syndrome Undiagnosed --> Reflex sympathetic dystrophy

List the 5 major sources of blood loss during traumatic injury. What type of traumatic injuries are at most risk for injury to one or more of these sources?

Chest, abdomen, pelvis (Retroperitoneum), femur + floor Blunt trauma

When is IV fluid administration indicated in the treatment of burns?

Children with >10% total body surface area and adults with >15-20% total body surface area burns Urinary .5-1 ml/kg/hr

49-year-old female with a 2-day history of right-upper-quadrant, colicky abdominal pain, as well as nausea and vomiting. Patient states she has had several episodes that wake her from sleep at like 2am in the morning, but typically only last an hour or so. She states this pain just won't go away. Examination shows significant pain with palpation in the right upper quadrant. What is your most likely? What is the name of the physical exam finding?

Cholecystitis Murphy's sign

Picks oranges and is having urinary and fecal incontinence. He's diaphoretic and had a seizure while out in the field.

Cholinergic toxidrome

When evaluating a hand injury, how will it present if there is an injury to the ulnar nerve?

Claw hand Numbness/ pain over 1.5 fingers

Flail chest is defined as: A. Fracture of the sternum from the chest wall B. Avulsion of the skin across the chest wall C. Penetration through the chest wall cavity D. 3+ ribs are fractured in at least 2 places

D. 3+ ribs are fractured in at least 2 places --> Paradoxical breathing

What is the GCS score in a patient who was injured in a motor vehicular accident and noted to be flaccid, with eyes opening only to pain, and who is only moaning? A. 4 B. 6 C. 3 D. 5 E. 7

D. 5

What is the GCS score of a patient who opens his eyes to voice, moans, and withdraws to pain? A. 11 B. 8 C. 10 D. 9 E. 7

D. 9

Which of the following medication classes is the recommended treatment for patients who have an anterior wall myocardial infarction with poor left ventricular function? A. BB B. CCB C. Potassium sparing diuretics D. ACEI

D. ACEI BB --> Negative inotropes

A 23 YOF who has been otherwise healthy presents with acute onset of severe LLQ pain. LMP 28 days ago. BP 90/70, pulse 120, temp 98.6F Hgb WNL, lactic elevated Bedside ultrasound was performed and there is fluid noted in the splenorenal recess. Formal ultrasound is pending. There are two diagnoses: _______ secondary to one of the below findings. Be specific. a. Mesenteric Ischemia b. Ovarian Torsion: Septic c. Ectopic Pregnancy d. Ovarian cyst rupture

Hemorrhagic shock secondary to ectopic pregnancy

Inflammation of the cornea (Keratitis) caused by a viral infection This virus is common and contagious. This virus cannot be eradicated as it establishes latency within the peripheral sensory ganglia and persists in the host for a lifetime. Symptoms: Patients will report eye redness, eye pain, blurred or decreased vision, photophobia, difficulty opening eye because of pain and irritation. What is the consequence of missing this diagnosis? Note: this infection can occur from injury (wearing contacts too long), infection, and disease.

Herpatic keratitis Blindness, can spread to optic nerve Ulcer

A 60 YOF presents with with altered mental status. She has a history of hypertension, diabetes type 2, Crohn disease, and dyslipidemia who has been off her medications x 1 week, but could not get in for a refill for 2 wks Meds: lisinopril 20 mg, metformin 1gm BID, atorvastatin 20 mg and prednisone 5mg daily. temperature 105F heart rate: 149/min blood pressure: 70/40 mmHg respiratory rate: 20/min O2: 90% on room air. Physical examination is remarkable for significant diffuse abdominal tenderness. Laboratory data demonstrates hyponatremia, hypercalcemia, normocytic anemia with eosinophilia. BG? 60

Adrenal crisis

What are the components of the primary airway survey?

Airway, breathing, circulation, disability, exposure/ environment

Which of the following will be affected by liver disease and how so. Albumin: AST: ALT: PT: INR: PTT: Plts:

Albumin: Low AST: Elevated ALT: Elevated PT: Prolonged INR: Prolonged PTT: Prolonged Plts: Decreased

An abrupt, transient localized swelling of deep skin layers or mucous membranes that most often affects the face, periorbital area, lips, larynx, or gastrointestinal tract. What might initial symptoms present like? secondary to

Angioedema Histaminergic release (from histamine): Annaphylaxis Nonhistaminergic (hereditary, drugs): Bradykinin mediators: ACEI

How do you calculate anion gap?What is normal, what is elevated?

Anion gap= Na+ - (Cl- + HCO3-) Elevated >12

What neurologic difference may you see if a patient has elevated pressure in their anterior compartment vs lateral compartment vs deep posterior compartment?

Anterior: Swollen, tender muscle at middle and distal ⅓ of the leg, pain worse with passive plantar flexion, decreased 2 point discrimination Lateral: Superficial peroneal nerve injured so muscles won't be able to produce eversion Deep posterior: Tibial nerve so muscles won't be able to plantarflex

What is the only toxidrome that results in hyperthermia with dry skin and decreased bowel sounds?

Anticholinergic

What is the key difference between anticholinergic and sympathomimetic toxidromes?

Anticholinergic= Dry Sympathomimetic= Diaphoretic

What is the #1 question you should ask a patient who has a head injury when considering their risk for an intracranial bleed?

Antiplatelets, anticoagulant use

When can pulse ox be inaccurate?

Anything that interferes with the transmission or absorbance of light: Darker skin, nail polish, dyshemoglobinemais, IV dyes, hypo perfusion, hypoxia (SpO2 <80%)

Disruption of the thoracic aortic medial layer provoked by intramural bleeding, causing separation of the aortic wall layers. How will this patient present? Symptoms (Depends where it is occurring): Sudden, severe, tearing chest pain, radiate up neck, radiate to back Physical exam findings: Dec peripheral pulses (Pulses paradoxes), HTN, diaphoretic,

Aortic dissection Ripping CP spreading to back radiating up neck, HTN, pulses paradoxes, diaphoretic

Hx of SSD. Hgb low and reticulocyte is zero

Aplastic crisis

Which condition can be caused by parvovirus?

Aplastic crisis

Which condition will result in anemia without a reticulocyte response?

Aplastic crisis

14 YOM presents to ED with periumbilical pain which started this morning. He denies fever, chills, nausea or vomiting. Denies hunger and states he has not eaten today. On exam you will palpate _________ to further evaluate your suspicions. If this area is non-tender, which 2 other exams can you perform to further evaluate your suspicions and how do you perform them.

Appendicitis RLQ (McBurney'so int) Psoas: Hip extension Rovsing: Palpate LLQ --> Pain on right Obtruator: Bend knee and twist internally

Describe the techniques to manage an external hemorrhage

Apply pressure --> Pack the wound --> Apply a tourniquet --> 2nd tourniquet

What is the differential for an anion gap metabolic acidosis? Name 2 conditions beside DKA in which you will see an anion gap metabolic acidosis.

High anion gap: Methanol, Uremia, DKA, Propylene glycol, Isoniazid intoxication, Lactic acidosis, Ethanol ethylene glycol, Salicylates Normal anion gap: GIT HCO3 loss: Diarrhea, external fistula Renal HCO3 loss: Proximal RTA, distal RTA, hyperkalemic RTA

A 66-year-old female who is obese presents to the emergency department unresponsive. The physical exam reveals sunken eyes and decreased skin turgor. A urinalysis is significant for large amounts of glucose with no ketones or proteins. POCT BG 800, remainder of labs are pending. Her unresponsiveness may be related to WHAT

Hyperosmolar hyperglycemic state

Inherited genetic disorder resulting in an obstructed outflow tract Sudden death is the biggest risk Most commonly presents with dyspnea followed by chest pain

Hypertrophic cardiomyopathy

Collection of blood in the anterior chamber of the eye resulting from blunt trauma or spontaneous which can occur in the setting of sickle cell disease or other increased bleeding states --> Inc IOP 2ry to mechanical obstruction of trabecular meshwork by the clotted blood What is the consequence of missing this diagnosis?

Hyphema Glaucoma

What are the electrolyte findings that may occur with an Adrenal crisis and why?

Hyponatremia: Cortisol deficiency --> Inc secretion of ADH and inability to excrete free H2O Hyperkalemia -->Aldosterone increases urinary potassium secretion so dec secretion --> Increased retention of potassium --> Peaked T waves

What is a late sign of volume depletion?

Hypotension

80 YOM with a history of hypertension, diabetes, hyperlipidemia, hypothyroid, chronic kidney disease, obesity and chronic constipation presents to ED with his wife who reports progressively worsening lethargy over the last month. She states her husband is a stubborn ole bitty and never takes his meds. I can't even get him into the doctor's office...ugh! BP 100/70 Pulse 50 Temp 95F BG 60

Hypothyroid crisis (Myxedma)

What are the 5 causes of hypoxemic respiratory failure ?

Hypoventilation Low FiO2 or low inspired PO2: High altitude Diffusion abnormality: Thickening of alveoli-capillary membrane V/Q mismatch: Ventilation/ Perfusion Shunt

What is a FAST exam and when is it indicated?

Identities free intraperitoneal or pericardial fluid in blunt trauma its Indications: Blunt and penetrating cardiac and chest trauma, trauma in PG, pediatric trauma, undifferentiated HOTN

Cauda equina can cause urinary incontinence. Why? How is this normally suppressed? If that innervation is lost, what else may present?

It causes the nerves that control the anal and urinary sphincter to be compressed High speed MVC and lower back fracture Saddle anesthesia, uni/bilateral leg radiation, decreased anal spinchter tone → No anal wink

Constrictive pericarditis: JVD, trachea, lung sounds, heart sounds, LE edema

JVD: Increaased, +Kussmaul sign Trachea: Midline Lung sounds: Clear, crackles Heart sounds: Pericardial knock LE edema: Yes

Tension pneumo: JVD, trachea, lung sounds, heart sounds, LE edema

JVD: Increased Trachea: Deviated Lung sounds: Unilateral absent breath sounds Heart sounds: RRR LE edema: None

Tamponade: JVD, trachea, lung sounds, heart sounds, LE edema

JVD: Increased Trachea: Midline Lung sounds: Clear Heart sounds: Distant LE edema: None

How should you open the airway of a patient with a cervical injury?

Jaw thrust maneuver

What is the name of the C1 fracture that typically results from axial loading?

Jefferson's

The nurse has given your 78-year-old patient morphine for her femur fracture, her bp is 90/70 from the blood loss, you need to reduce the fracture and would like to give IV sedation before doing so. You ask the nurse to grab midazolam, but she looks at you like... really? Why does the nurse not want to give this medication and which medication can you use instead?

Ketamine Miadazolam CI in hypovolemic patients

How do you treat acute heart failure? Why should you avoid beta blockers? How could BiPAP help?

LMNOP = Lasix (Furosemide) → removes fluids, Morphine → reduces preload reducing heart strain, Nitrates (Venodilators) → reduce preload and afterload, Oxygen, Position → place upright to decrease venous return Beta adrenergic receptors vasodilate vessels and increase HR. they are CI in acute HF due to acute negative inotropic effects → could decrease contractility and HR, blunts tachycardia BiPAP: Helps increase intrathoracic pressure causing increased preload and thus increase cardiac output.

What are some common mechanism of a Lis franc injury and how does it present?

Large pulling force on foot, hyperplanar flexed forefoot Not able to bare weight, pain on plantar aspect

Limping child with a necrotic femoral head

Legg-Calve-Perthes

What are the two most common abdominal organs to be injured following blunt trauma?

Liver and spleen

Who is at risk of having their airway comprised? What type of trauma?

Loss of neurogenic innervation of the upper airway musculature: Brainstem injury, distal nerve dissection Upper airway obstruction or injury: Facial trauma, blunt trauma to the neck, penetrating neck injury

What are the distinguishing features of this type of shock as compared to others (2)?

Low blood glucose: Cortisol deficiency Will not respond to fluids: Flows through kidneys without aldosterone

66 YOM presents to ED after accidentally electrocuting himself while working on his outlets at home. He states he is normally healthy and takes no medication. The affected area is currently unremarkable, but you noticed his pulse is 55bpm. You check his most recent outside records and his pulse at that time just a week ago was normal. Did he experience injury from low or high voltage? What arrhythmia is most commonly seen with this type of voltage injury?

Low volatge Vfib

What MAP and what is the minimal adequate MAP for organ perfusion?

MAP= (SVR) (CO) 65-70

Select the most common cause of injury-related death in patients 65-74 years of age. A. Suicide B. Toxic ingestion C. MVA D. Falls

MVA Over 75= Falls

What is biPAP and when is it used?

Maintains alveolar pressure and prevents the alveoli from collapsing COPD exacerbation, acute heart failure, chest trauma, obesity hypoventilation syndrome

A spiral fracture of the fibula due and a high ankle sprain is called what?

Maisonneuve fracture

A 46-year old man presented at the ED with hematemesis following excessive retching and vomiting after having alcohol binge. He's hemodynamically stable and there is no fever. What is the most likely diagnosis?

Mallory Weiss tear

What are 3 ways you can improve airway potency prior to placing a definitive airway?

Manually: Head tilt or jaw thrust Oropharyngeal airway Nasopharyngeal airway

6 YOF with otalgia, worsening over the last 5-days. The patient is afebrile. Examination reveals edema of the external auditory canal producing an anterior and inferior displacement of the auricle with percussion tenderness posteriorly. More common in young children and infants. What complication can occur if this diagnosis is missed?

Mastoiditis Hearing loss, abscess, meningitis

An 18-year-old presents to the emergency department by EMS after being involved in a motor vehicle collision. She was the unrestrained driver, roll-over on freeway, no airbag due to age of car, spider-webbing of windshield, and broken steering wheel. Car suffered major intrusion with extrication time of twenty minutes. What plays a vital role in anticipating the injury patterns and treatment of this patient? A. Age of patient B. Mechanism of injury C. Roadway conditions D. Body mass index (BMI)

Mechanism of injury

34-year-old man who is brought by his wife because she believes her husband is very ill. The patient initially had a headache that progressed to neck stiffness and an inability to look at bright lights. His temperature is 103.1° F, blood pressure is 134/82 mmHg, and respirations are 20/min. Extreme pain is elicited upon flexion of the patient's neck and the patient's legs.

Meningitis

A 63 YOF with a history of atrial fibrillation (coumadin on hold) status post lumbar fusion reports acute severe abdominal pain. WBC, CRP and Lactic acid are elevated CT abdomen and pelvis with IV contrast is notable for pneumatosis intestinalis and pneumatosis portalis

Mesenteric ischemia 2ry to clot from afib/ post surgery

32 YOF with a history of steroid dependent lupus presents with report of abdominal pain and distension x 1 week. She reports pain to abdomen during the ride to the hospital every time her husband hit a pothole and states the pain is now just intolerable. Which complication is present and what is the most likely cause?

Mesenteric ischemia or bowel perforation Steroids + lupus

What is the expected pupillary finding for a patient who has overdosed on heroin (Opioids)? Miosis or mydriasis?

Miosis

Under what conditions can you discharge a patient from the emergency room who has had an anaphylactic reaction?

Monitor for 6-12 hours

You suspect your patient has an esophageal injury. You order a neck x-ray, but there is no evidence of injury. Is this enough to r/o esophageal injury? Why or why not. If your answer is no, what do you need?

NO does not rule it out You need esophagram and endoscopy

What is aldosterone normally responsible for? How will your patient present if they are having an acute deficiency of this hormone?

Na+ reabsorption, K exertion, regulates BP and blood volume Deficiency: HOTN, high pulse

60 YOF with a history of steroid dependent COPD presents with report of progressively worsening rash to right foot that started as large blisters and now her skin is turning black.

Necrotizing fasciitis

Red, swollen, painful leg with air on an x-ray

Necrotizing fasciitis

Waterhouse-Friderichsen Syndrome is typically caused by which bacteria? How does this affect the adrenal glands?

Neisseria Adrenal hemorrhage

Which is the only shock that does can develop hypotension, but does not have a narrow pulse pressure or tachycardia and why?

Neurogenic shock Pulse pressure represents blood volume and tachycardia represents body's compensatory response

A patient is taking Haldol and Librium to treat their psychiatric illness. Which syndrome are they most likely to develop and what is the characteristic muscle finding for this syndrome?

Neuroleptic malignant syndrome Lead pipe rigidity

How would a patient with a complete airway obstruction present?

No sound

Severe infection of the fat and muscles posterior to the orbital septum. May occur due to spread of infection from adjacent structure like sinuses, teeth and lacrimal glands. Most cases are bacterial and occur in children (staph, anaerobic) Fungal infections are less common but can be typical for DM or immunocompromised patients. So, consider this when infection is not improving on antibiotics. What physical exam findings are you looking for? What are the complications that may occur if this is mis-managed or missed?

Orbital cellulitis Pain with eye movement, proptosis, erythema, vision loss, afferent pupillary defect Optic nerve damage, meningitis, abscess, cavernous sinus thrombosis

A 17-year-old male presents with 2 months of lower extremity pain that is worse at night. The patient states he first noticed the pain following an injury during a high school football game. Radiographs show a bone tumor with deposition of calcified osteoid matrix and an associated sunburst periosteal reaction.

Osteocaroma --> Sunburnt appearance METS to lungs

No injury to the bone, elevated CRP

Osteomyelitis

When should you order a CT in a pediatric patient?

PECARN

10YOM states he was hit in the face by a baseball at practice 3 days ago. He was seen in ER at that time, but his x-rays were negative for fracture. He feels like it's getting harder and harder to breath out of his nose with report of feeling pressure within the nares. What is this patient at risk for developing and what is the consequence of missing this diagnosis?

Septal hematoma from epistaxis Loss of cartilage --> Saddle nose deformity

Tramadol and Celexa will increase risk for?

Serotonin syndrome

Limping child with a slipped femoral head

Slipped capital femoral epiphysis

32 YOM with a history of Crohn's disease s/p resection 3 years ago presents to ED with report of abdominal pain, distension, nausea and vomiting. Patient states he hasn't had a bowel movement or flatulence in 1 week. What is your most likely diagnosis? What are your expectations for bowel sounds? early vs late

Small bowel obstruction Early: Increased Late: Decreased/ none

How would a patient with a retropharyngeal abscess present?

Sore throat, trouble swallowing, no uvula deviation, muffled voice, stiff neck

1 YOM with a history of sickle cell disease presents to ED with mother who reports inconsolable crying. On exam he appears pale, his heart rate is tachy, but regular and his spleen is palpably enlarged. Mom states his baseline hgb is 9 g/dl. Current Labs Hgb 6.2 g/dl (11-14) Reticulocyte pending Why is this happening?

Splenic sequestration syndrome

How would a patient with an airway obstructed by edema or a FB present?

Stridor

Which presents on CT as a crescent (Concave) shaped bleed?

Subdural

If your patient has a lateral ankle injury, which nerve may be injured and how will this present?

Sural nerve: Numbness/ pain along lateral ankle

Hallucinating, on cocaine, temp 103F What do his pupils look like?

Sympathpmimetic --> Mydriasis

How is HOTN defined if pt is HTN at baseline? Systolic drop Drop in MAP

Systolic drop by 20 mmHg from baseline Drop in MAP > 40

Define HOTN. Is it an early or late sign of shock? Systolic Diastolic MAP Pulse pressure Orthostatic SBP Orthostatic DBP

Systolic: <90 Diastolic: <60 MAP: <65 Pulse pressure: <20 Orthostatic SBP: Decreases by 20mmHG Orthostatic DBP: Decreases by 10mmHG Late finding: Body can compensate for up to 30% blood volume loss

Heavy period, uses maximum super tampons. Has fever and HOTN

TSS

What is the earliest sign of volume depletion?

Tachycardia

Adrenal crisis commonly occurs during an acute GI infection. Why?

Vomiting and diarrhea --> Decreased blood volume --> Body releases aldosterone (Inc Na+ absorption which inc H2O absorption) to try and compensate --> Body doesn't have it so can't compensate

When evaluating a hand injury, how will it present if there is an injury to the radial nerve?

Wrist drop Numbness/ pain over posterior arm/ forearm, below the fingers of the medial 3.5 fingers

Which bullous rash will not have a positive Nikolsky sign?

Erythema multiforme Bullous pemphigoid

What is the appropriate preventive treatment in a patient with an acute full thickness, circumferential burn to the arm?

Escharotomy

Your patient is having an adrenal crisis and is now crashing. The nurses are grabbing the intubation cart. You suspect this patient will remain intubated at least overnight and thus you will need a continuous infusion of your induction agent. Which agent should you avoid and why?

Etomidate --> Will cause adrenocortical suppression Note: a single induction dose blocks the normal stress-induced increase in adrenal cortisol production for 6-8hrs up to 24hrs in elderly and debilitated patients. this is the number one reason why you cannot use this as a continuous infusion in ICU will increase mortality

Describe the indication for fluid resuscitation

Evidence of shock Active bleeding >10% blood loss in child, >15-20% blood loss in adult

Rhabdomyolysis can result in an acute kidney injury. What causes the acute kidney injury?

Excess myoglobin kidney disposition

Pt says I got an electrical shock to hand. Small amount of erythema on finger. What should you look for?

Exit wound

What are the criteria for diagnosing acute chest syndrome?

Fever, hypoxia, pulmonary infiltrates

Bacterial cutaneous infection that results in rapid tissue destruction to the perineum is called what?

Fournier gangrene

What is the scoring system used to categorize head injury severity?

GSC 13-15: Mild head injury GSC 9-12: Moderate head injury GCS 3-8: Severe head injury

What is the goal of treatment for DKA? When can you discontinue the insulin drip?

Goal: Improve metabolic acidosis and close the anion gap Discontinue:Anion gap resolved

How would a patient with epiglottis or an impending airway closure present?

Grunting

How would a patient with an airway obstructed by vomit/ secretions present?

Gurgling

+ BP > 180/120 with end organ damage

HTN emergency

When evaluating a hand injury, how will it present if there is an injury to the median nerve?

Hand of benediction/ ape hand Numbness/ pain over first 3.5 fingers

What is the name of the C2 fracture that typically occur due to a combination of compression, hyper flexion, and hyperextension?

Hangman fracture

A 65YOM presented in the emergency department with altered sensorium, irritable behavior, and fever. His symptoms developed few hours after he went out to buy groceries in the hot weather. He reports walking 4 miles to and from the store. His PMH is unremarkable. Vitals: 104F, 120 bpm, bp 110/80 mmHg What do I have? What do you expect to find when you examine my mucous membranes?The elasticity of my skin?

Heat stroke Dry mucous membranes, poor skin turgor

What is the Grey-Turner's sign and what does it represent? Is it an early or late sign of injury?

Bleeding along the flank Retroperitoneal hemorrhage Late finding

What is Cullen's sign and what does it represent? Is it an early or late sign of injury?

Bleeding around the umbilicus Ruptured AAA, ectopic PG, blunt trauma with intraabdominal hemorrhage Late finding

What is hemotypmanum and what does it represent?

Blood in the tympanic cavity Basilar skull fracture

What is the most common mechanism of epidural hematomas? Which vessel is most commonly involved? Blunt trauma to which part of the skull is most vulnerable?

Blunt trauma to temporal region resulting in bleeding from middle meningeal artery

What is Cushing Reflex (Vasopressor response) ? Describe the mechanism

Bodies response to acute ICP HTN + Bradycardia + Irregular respirations or apnea

If you are unable to record a BP, what can you say your minimal systolic BP is if you can feel one of the following peripheral pulses?

Carotid: >60 Femoral: >70 Radial: >80 Dorsalis pedis: >90

20 weeks gestation or greater BP > 140/90 + proteinuria or BP > 140/90 + end-organ damage •Platelets <100K •Renal insufficiency Cr > 1.1 •Impaired liver function: LFTs > 2x normal •Pulmonary edema •Cerebral or visual symptoms

Preeclampsia

Anaphylaxis: You have a 60kg patient How many mL would you give? (1mg/1mL)

1 mL

What is cortisol normally responsible for? How will your patient present if they are having an acute deficiency of this hormone?

"Fight or flight" Deficiency: Fatigue, weight loss, dec appetite, HOTN, hyperpigmentation of skin, hypoglycemia

What are risk factors for ectopic pregnancy?

Previous ectopic, PID, invitro, adhesions, inc maternal age, smoking

Burn from hot water with blister and pain

2nd degree partial thickness

Hot oil, no blanching, no pain, no refill

3rd degree deep partial thickness

How long should you observe a pediatric patient in the ED if you don't get a CT scan?

4-6 hours to assess for changes in clinical status

47 YOM lost while hiking in the mountains in below freezing weather was lost for 3 days and presents via rescue team with this injury. His finger tips are black and fall off

4th degree frostbite

A 5YOF presents to ED after a near-drowning accident. Her mother was nearby and states she believes she was submerged for less than a min. She is currently asymptomatic, physical exam is unremarkable, vitals are stable with o2 saturation of 98% on RA and her CXR is normal. How long must you observe the patient for before they can be discharged home? Why must you wait this period of time?

6-8hrs Respiratory distress typically develops within this time

What is an abnormal blood pressure level?

<90/60 or <baseline or change of 20mmHG w/o change in postion

What is an abnormal capillary refill time?

>2 seconds

A patient is brought to the emergency room with acute onset of dyspnea and tachypnea. He has a long history of alcoholism and was involved in a motor vehicle accident two days ago. He is hypoxic with crackles auscultated bilaterally. Chest radiography reveals diffuse bilateral infiltrates which spare the costophrenic angle and air bronchograms, there was no cardiomegaly or pleural effusion noted. Oxygen saturation is 70%. Which of the following is the most important initial treatment? A Tracheal intubation B. Bilateral chest tube insertion C. Type-specific packed cells D. Colloid solutions What type of hypoxemia is this? From 5 types of hypoxemia.

A Tracheal intubation --> ARDS Diffusion impairment

When should you suspect a cervical injury?

A patient found unconscious with no witness

A 46 YOF with a recent cholecystectomy presents with fever, RUQ pain and jaundice. Her labs show elevated LFTs, leukocytosis with left shift and an elevated bilirubin. What is the most likely diagnosis? A. Choledoholithasis B. Cholangitis C. Acute cholcystitis D. Hepatitis A

B. Cholangitis

What is the sensitivity of the Canadian CT Head Rule for injuries requiring neurosurgery? A. 100% B. 62% C. 76% D. 82% E. 51%

A. 100%

What type of amnesia is commonly seen in TBI? A. Anterograde B. Retrograde C. Lacunar D. Dissociate E. Transient global

A. Anterograde

Which of the following animals is the major cause of human rabies in the United States and, therefore, poses the highest risk? A. Bats B. Rabbits C. Rodents D. Dogs

A. Bats

Patient presents to the ED with fever/chills, RUQ pain, AMS, shock. Which of the following diagnostic findings would be consistent with the patients presentation? A. Biliary tract infection 2ry to obstruction B. Gallstone found in gallbladder C. Gallstone causing infection in cystic duct D. Gallstone in the common bile duct causing ductal dilation

A. Biliary tract infection 2ry to obstruction

Which of the following diagnostic findings is the gold standard for identifying cholangitis? A. Cholangiography displaying the obstruction B. Elevated LFTs C. Leukocytosis D. Abdominal ultrasound displaying dilation of the common bile duct

A. Cholangiography displaying the obstruction

What medications are used in combination to lower anxiety and blood pressure in emergent situations? A. Diazepam and nitroprusside sodium B. Midazolam and nitroglycerin sublingual C. Alprazolam and enalapril maleate D. Buspirone hydrochloride and Metoprolol succinate

A. Diazepam and nitroprusside sodium --> HTN emergency Nitroprusside= Potent atrial dilator

A 16-year-old male presents to the emergency department with complaints of fever, headache, and myalgias for the past four days. Yesterday, he developed abdominal pain, nausea, and had three episodes of vomiting. The patient was recently diagnosed with appendicitis, and an appendectomy was performed a week ago. He mentions that he has been experiencing mild pain in the region of the stitches that improves with ibuprofen. Vitals show a temperature of 38.8 Celcius, respirations of 17 breaths/min, pulse rate of 103 beats/min, and blood pressure of 80/65 mmHg. The patient appears restless and agitated. Which of the following may be seen on the physical examination of this patient? A. Diffuse, confluent erythematous rash on the trunk and extremities B. Skin desquamation on the trunk and extremities C. Target like lesions on the trunk and extremities D. Vesicular lesions on the trunk and extremities

A. Diffuse, confluent erythematous rash on the trunk and extremities

A 24-year-old male presents to the office with multiple rashes on his palms and soles. There is mild itching associated with the rash. He denies any fever, recent drug use, or recent travel. He is sexually active with his girlfriend and uses a condom for contraception. On examination, the vitals are stable. There are multiple papular, target lesions symmetrically present on the bilateral palms and soles. There is no such lesion on other mucosal surfaces. Skin biopsy shows perivascular lymphohistiocytic infiltrate present in the superficial dermis without necrotic vascular lesions. Infection with which of the following organism is most likely the cause of this condition? A. HSV B. HIV C. HPV D. Chlamydia trachomatis

A. HSV --> Erythema multiforme

A patient involved in a minor motor vehicle crash is brought to the emergency department by a family member who was riding in the car but was unhurt. The family member states that the patient was unconscious for about 2 minutes but seems "okay" now. What diagnostic procedure would be most helpful in assessing this patient? A. Head CT scan B. LP C. Skull radiographs D. Electroencephalogram

A. Head CT scan → LOC possible Epidural hematoma, skull fracture, edema, Lumbar puncture → Blood from subarachnoid hemorrhage Skull radiographs → Fractures Electroencephalogram → If pt said person was unconscious and had shaking/ tongue bite or was incontinent

Which do you give first when treating DKA? Why? A. IV saline B. Sq insulin C. IV saline D. IV bicarb

A. IV saline

A 40-year-old G6P5 at 18-weeks of gestation presents to the emergency department with a complaint of three episodes of seizures that occurred fifteen minutes ago. The patient also complains of on and off severe headaches for the last four weeks. On further questioning, she reveals that she has episodes of blurry vision as well. A detailed history reveals that her previous two pregnancies were diagnosed as a case of pre-eclampsia. Her vital signs show blood pressure 170/80 mmHg, heart rate 96 beats per minute, respiratory rate 19 breaths per minute, and temperature 37 C. A urinalysis performed shows proteinuria. What is the most appropriate initial step in the management of this patient? A. Magnesium sulfate only B. Magnesium sulfate and lorazepamc C. Magnesium sulfate and diazepamd D. Magnesium sulfate and haloperidol

A. Magnesium sulfate only --> Eclampsia, non active seizures Active --> Lorazepam + Mangesium Mangisum OD --> Diazpam CI --> Haloparadol

An elderly appearing adult male patient is transported to the emergency room with unconsciousness for an underdetermined amount of time. There is no family and the only history is provided by the paramedics. The patient arouses to verbal and painful stimuli. VS: T-97.0 degrees F rectally, P-52 bpm, R-10, BP-95/60 mmHg. Physical examination is unremarkable except for ecchymosis across his extremities. A Foley catheter is inserted draining a small amount of dark brown urine. Urine dipstick reveals 4+ positive hemoglobin and protein. Microscopic urinalysis reveals no RBCs but many renal tubular epithelial cells and renal tubular casts. Drug screen is negative, blood alcohol is 2.5 mg/dL, and creatinine is 4.9 mg/dL. What is the most likely diagnosis? A. Rhabdomylsis causing acute renal failure B. Obstructive uropathy causing acute renal failure C. Ethanol ingestion causing acute renal failure D. Methanol ingestion causing acute renal failure

A. Rhabdomylsis causing acute renal failure

What needs to be done for a placental abruption? A. Stabilize maternal circulation B. Move to the operating room before getting a fetal monitor tracing C. Administer oxytocin after placing an intrauterine pressure catheter D. Try to deliver vaginally even in the presence of non-reassuring fetal tracing

A. Stabilize maternal circulation Vaginal bleeding, uterine tenderness on palpation, abnormal fetal heart tones

A foreign body lodged in the trachea that is causing partial obstruction will most likely produce what physical examination finding? A. Stridor B. Aphonia C. Inability to cough D. Progressive cyanosis

A. Stridor All others are signs of complete obstruction

A 42-year-old female presents to the emergency department with persistent epigastric abdominal pain for the past 10 hours while she was at a barbecue party. She also complains of nausea and bloating. The patient states experiencing similar symptoms in the past; however, they were self-resolved. All laboratory results were normal. Abdominal ultrasound revealed a thickened gallbladder wall with pericholecystic fluid, gallstones, and positive murphy's sign. Which of the following is the best treatment for this condition? A. Surgery within 24-48 hours of admission B. IV abx and surgery within 1 week C. Surgery as an emergency D. Elective surgery in 4 weeks

A. Surgery within 24-48 hours of admission --> Uncomplicated Cholecystitis Complicated (Septic --> Inflammation around gallbladder) --> PCT and empiric abx

What is the most common and the most serious pulmonary life threat observed in trauma patients? A. Tension pneumothorax B. Open pneumothorax C. Atelectasis D. Flail chest E. Massive hemothorax

A. Tension pneumothorax

A 28-year-old male presents with burns sustained from hot grease splashed on his left hand earlier this afternoon. The burn extends from his palm to the volar aspect of his wrist and has an erythematous base, covered by an intact blister. There are a few small scattered blisters over the dorsum of the left hand. Which of the following is the initial intervention of choice? A. Tetanus prophylaxis B. Admission to a burn unit C. Intravenous fluid administration D. Debridement of blisters

A. Tetanus prophylaxis

A 20 YOF comes into the ED with severe RUQ pain after eating a nice dinner with her boyfriend at panda express. She said that the pain is so severe that she threw up on the way over to the ED. Her labs are WNL. What would the best initial diagnostic test be? A. Transabdominal US B. CT scan C. Exploratory lapratomy D. MRI

A. Transabdominal US

A 16-year-old is involved in an MVA and is intubated at the scene. He is rushed to the emergency department where the trauma team immediately begins to resuscitate him. His blood pressure is 69/45mmHg, and his heart rate is 138 beats/minute. He has 2 large-bore IVs running. Which of the following positions should you place the patient in to improve blood pressure? A. Trendelenburg B. Lithotomy C. Simms D. Reverse Trendelenburg

A. Trendelenburg: Head down, feet elevated

A 16 year old male presents to the ED with his mother 3 days after removal of his wisdom teeth. The mother states that the child has found it difficult to control his pain and now is bleeding excessively from his would and his nose. PT and INR are elevated and LFTs are twice the upper limit of normal? What toxicity is this patient experiencing? A. Tylenol B. Salicylate C. Opioid D. Hypnotic

A. Tylenol

A 20-year-old male football player comes into the ER complaining of RUQ pain today. Further questioning reveals that yesterday, he had trouble eating anything at all (he just had no desire). You ask if he's on any medications, which he denies, other than occasional over the counter pain relief that he takes for muscle aches related to his athletics. ABG shows pH= 7.20, pCO2=40 and HCO3= 16, LFTS are elevated and PT/PTT time is prolonged A. Tylenol B. Aspirin C. Ibuprofen D. Alcohol

A. Tylenol

You are working in the ED and pick up the next chart. It is a 40-year old woman with the following vital signs: T38, BP 101/60, HR 130, RR 20, SpO2 92% on RA. The triage nurse notes that the patient had a fever for 2 days, productive cough and is now slightly confused. She has no other comorbidities. What is the next best step? A. You initiate fluids and tell the nurses to draw a septic work-up. You see her right away. Based on the NEWS score, she is likely to be suffering from occult sepsis. B. You ask the nurses to draw a CBC. You will then use the WBC to calculate a SIRS score as she currently does not satisfy the requirement yet to initiate sepsis management. C. Based on the qSOFA score, she is not high risk so you will see another patient first. D. You ask the nurse to draw a lactate. That is the best test for diagnosing occult sepsis. What are SIRS criteria and how do you decide if a patient has sepsis? Septic shock?

A. You initiate fluids and tell the nurses to draw a septic work-up. You see her right away. Based on the NEWS score, she is likely to be suffering from occult sepsis. SIRS: 2 of the following 4 criteria Fever >100.4°F or hypothermia <96.8°C Tachycardia >90 beats/minute Tachypnea >20 breaths/minute Leukocytosis >12 or leucopoenia <4

65 YOM with a history of hypertension, diabetes, and tobacco dependence presents with chest pain while shoveling this afternoon. What is your do not miss? What additional symptoms would increase the likeliness of this diagnosis?

ACS Diaphoresis, N/V, radiation to left arm/ jaw

When should you order a CT in patients >65?

ALWAYS high risk

Engorgement of corpora cavernosa in the absence of sexual arousal that is prolonged and frequently painful. What causes this? What is the most common cause in children?

Priapsrism Ischemic Sickle cell, meds (Flomax for BPH, hydroxyzine)

Tonometer pressure is 28 mmHG to the left eye with a afferent pupillary defect indicates what?

APD --> Optic nerve damage Acute close angle glaucoma

Diffuse pulmonary infiltrates on CXR

ARDS

3 YOM with sickle cell disease presents with report of rhinorrhea and dry cough x 2 days. On exam you hear rales on the right and his temp is 100.8F. He appears to have conversational dyspnea and his o2 saturation is 91% on room air. What emergency do I have? Is tactile fremitus increased or decreased?

Acute chest syndrome Increased

A mother presents to the ED concerned that her 3 year old son is profusely sweating and has had urinary incontinence for the past 12 hours. The mother states she thinks he might have gotten into her cigarettes. What toxidrome does this child have? A. Anticholinergic B. Cholinergic C. Opioids D. Sympathomimetic

B. Cholinergic

An 83-year-old female is brought into the emergency department after being found in her home by her neighbors who say she is not acting right. When questioned, she is confused and minimally arousable. Her neighbors say the air-conditioning in the building has been broken the last few days. Despite feeling warm, she is not sweating, and there is vomit on her sweater. Which of the following distinguishes heat stroke from heat exhaustion? A. Rectal temperature above 104f B. Confusion and mental status changes C. Vomiting and sign of dehydration D. Anhidrosis

B. Confusion and mental status changes

At what temperature does exposed skin begin to be at risk for frostbite? A. 23F B. 14F C. -4F D. -13F

B. 14F

Prone position for ventilation is used in patients with which of the following disorders? A. Pneumothorax B. Acute respiratory distress syndrome C. Sepsis D. Diaphragmatic hernia

B. Acute respiratory distress syndrome --> Improve functional residual capacity and postural drainage

A 35-year-old female presents to the emergency department complaining of the acute right upper quadrant (RUQ) pain onset last night. The patient states pain started after eating fettuccini alfredo. The patient has taken antacid medication without relief. She is otherwise healthy, and only past surgical history is 4 prior cesarean section. The patient's white blood count is 18,000 mm3, alkaline phosphatase (ALP) 55 IU/L, alanine aminotransferase (ALT) 30 IU/L, aspartate aminotransferase (AST) 40 IU/L, total bilirubin 1mg/dL, and lipase is 105 IU/L. An abdominal ultrasound is negative. Which of the following statements best represents the best disposition for this patient? A. Her elevated enzymes are representative of a fatty liver. Discharge and have her follow up with PCP to check cholesterol levels and provide weight loss management. B. Admit patient to further evaluate the findings with a hepatobiliary iminodiacetic acid (HIDA) scintigraphy C. Recommend outpatient evaluation with a general surgeon for further evaluation

B. Admit patient to further evaluate the findings with a hepatobiliary iminodiacetic acid (HIDA) scintigraphy --> Check for cholecystitis, persistent pain + elevated WBC + Elevated liver enzymes you can't go home even if US is normal

A 32 YOM is brought to the ED by his wife because he's been acting psychotic. She doesn't know if he's taken anything. He seems agitated and disoriented. His skin is dry and flushes. His pupils are measured at 7mm. There are decreased bowel sounds. His blood pressure is 130/90 mmHG, pulses rate is 100 bpm, RR 14, and temperature 100.4F. What is the toxidrome being described? A. Serotonin syndrome B. Anticholingergic C. Sympaothmimitmetic D. Cholingergic

B. Anticholingergic

A 35-year-old comes in an ambulatory clinic established for the underserved population. He complains of pain along the lateral aspect of his right foot. On further questioning, he states that he keeps wearing the same socks for many days. Vital signs are heart rate is 78/min, blood pressure 118/65 mmHg, respiratory rate is 14/min, and temperature 98.6F (37 °C). Physical examination reveals the cold and blotchy skin of the right foot. There is decreased sensation in his right foot. What is the next step in the management of this patient? A. Applying moisturizer B. Applying warm packs C. Prescribe antibiotics D. Topical antifungal cream

B. Applying warm packs --> Immersion syndrome aka Trench foot

A patient is stabbed in the back and presents to you in the trauma bay. How would you quickly assess if the patient has an intact airway? A. Listen over their mouth and nose for breathing sounds B. Ask the patient to say their name C. Perform a jaw thrust procedure D. Look to see if the patient's chest is rising and falling with respiration E. Perform an upper endoscopy

B. Ask the patient to say their name

A 30-year-old patient is stabbed in the back. The patient is alert but complains of difficulty breathing. The patient is intubated en route to the hospital. When this patient presents to you in the trauma bay, what are your next steps in management? A. Obtain CT, ultrasonography, or aortic imaging studies of the chest B. Assess airway, breathing, and circulation C. Assess for signs of shock and obtain a blood pressure D. Obtain a chest x-ray and pelvis x-ray E. Assess breathing and circulation

B. Assess airway, breathing, and circulation

The bite from which of the following is associated with fever, lacrimation, rhinorrhea, bradycardia, hypertension, and tachyarrhythmias? A. Brown recluse spider B. Black widow spider C. Killer African bee D. Wasp

B. Black widow spider

Which of the following is true regarding electrical injuries? A. Blood vessels and nerves offer the least resistance to electrical conduction resulting in the greatest amount of injury. B. Bones and fats offer the greatest amount of resistance to electrical conduction resulting in the greatest amount of injury C. Resistance to conduction does not determine the extent of injury, but the degree of external cutaneous burns caused by electrical injury predicts the level of internal tissue damage.

B. Bones and fats offer the greatest amount of resistance to electrical conduction resulting in the greatest amount of injury

In a multiple-trauma patient, the screening x-rays that are usually obtained are which of the following? A. Pelvis, chest, and skull B. Cervical spine, chest and pelvis C. Abdomen, skull, and pelvis D. Chest, abdomen, and pelvis E. Cervical spine, abdomen, pelvis

B. Cervical spine, chest and pelvis 4+ floor: Chest, abdoment, pelvis, femur

A 45-year-old male is brought to the emergency department in a delirious state. His wife states that he recently had a viral illness, during which time he has stopped taking his regular medications. He regularly follows up with an endocrinologist. Vital signs are pulse rate 120/min, blood pressure 102/60 mmHg, respiratory rate 21/min, and temperature 102 F. Physical examination reveals a delirious patient with a rapid heart rate on auscultation. What is the next step in the management of this patient? A. Hydration, cooling blanket, and dopamine B. Hydration, cooling blanket, beta-blockers, corticosteroids C. Hydration, antipyretics, beta blockers, and benzodiazepines D. Hydration, antipyretics, and beta-blockers

B. Hydration, cooling blanket, beta-blockers, corticosteroids --> Thyroid storm

A 65-year-old male presents to the emergency department complaining of an acute onset of pain that started in the anterior chest. He says the pain is excruciating and now radiates between his scapula. The patient has a history of hypertension, coronary artery disease, and chronic obstructive pulmonary disease. On physical examination, he appears anxious and diaphoretic. His respiratory rate is 22 breaths/min, the pulse is 120 bpm, and blood pressure is 80/55 mmHg. Chest radiography shows widened mediastinum. What is the next step in management? A. Initiate thrombolytic therapy B. IV fluids followed by surgery C. Beta-blockers therapy followed by surgeryd. D. Measure cardiac enzyme levels

B. IV fluids followed by surgery --> Aortic dissection Hypotesnive

A 17-year-old male is accidentally struck in the right eye while playing football and is immediately transported to the hospital. In the emergency room, he complains of severe pain behind the eye as well as double vision. On examination, he has exophthalmos, cannot move his right eye upward and blood is noted in the anterior chamber. Which of the following is the most appropriate course of action at this time? A. Apply ice packs and cold compresses B. Immediately refer the patient to an ophthalmologist C. Attempt to keep the patient calm and order a skull x-ray D. Administer a dose of intramuscular broad-spectrum antibiotic

B. Immediately refer the patient to an ophthalmologist Blow out fracture

A patient presents with chest pain. ECG done in the emergency department reveals ST segment elevation in leads II, III, and AVF. This is most consistent with a myocardial infarction in which of the following areas? Which coronary artery is affected and what type of consequences may occur (i.e, change in rate, rhythm, function)? A. Anterior wall B. Inferior wall C. Posterior wall D. Lateral wall

B. Inferior wall --> RCA --> Bradycardia

A burn patient is currently under treatment in the emergency department. Initial fluid resuscitation is started. The burn involves epidermal and dermal layers involving the lower limbs and trunk area. In this patient, the fluids will shift to which space? A. Intravascular B. Interstitial C. No shifting D. Intracellular

B. Interstitial Burns --> Inc capillary permeability --> Fluid shifting from intravascular to interstitial space

A 44-year-old man is brought to the emergency department with a sore throat and high fever. He appears toxic and anxious. He attended a wedding four days ago where a groomsman "had a cold." He is visibly struggling to maintain a conversation but insists, "I'm fine." He is unable to handle his own secretions and is drooling and has inspiratory stridor and was seen in the emergency department last night for similar symptoms. He is sitting with his neck extended during this conversation. What is the most appropriate next step in the management of this patient? A. PO course of clindamycin B. Intubation C. Fine-needle aspiration D. Drainage of recurrent peritonsillar abscess

B. Intubation --> Epiglottis

A 35-year-old male presents to the emergency department after sustaining burns. He accidentally dropped a pail of boiling water on himself. His examination reveals superficial burns involving his right chest and partial thickness burns involving the anterior surface of the forearm and hand. The burns on the chest are dry, blanch with pressure, and are painful. While the burns on his forearm and hands are moist and weeping, are painful to touch and blanch with pressure. The area involved by the partial-thickness burn is calculated to be about 6% of the total body surface area. Which of the above patient parameters warrants a referral to a burn center? A. Percentage of body surface area involved B. Involvement of the hand C. Pain with palpation of wound D. Involvement of the chest

B. Involvement of the hand

A 35-year-old female presents to the emergency department with acute onset of shortness of breath and chest pain. Her past medical history is significant for hypertension, morbid obesity, and uncontrolled asthma requiring multiple admissions to the hospital. She was recently admitted to the intensive care unit for 5 days for an asthma exacerbation. Physical examination reveal hypotension, tachycardia, and tachypnea with hypoxia on ambient air. The pulmonary examination is unremarkable. Contrast-enhanced computed tomography confirms the diagnosis. The patient is started on heparin infusion and admitted to the intensive care unit. On day 3 of admission, she remains hypotensive and requires inotropic support. Laboratory workup reveals worsening thrombocytopenia. Echocardiography shows right ventricular dilation and flattening of the interventricular septum. She subsequently decompensates and goes into cardiac arrest. Which of the following could have prevented this outcome? A. Order IV heparin and monitor PTT B. Order for catheter-directed thrombolysis procedure C. Order endovascular thrombectomy D. Order Lovenox SQ 1mg/kg BID

B. Order for catheter-directed thrombolysis procedure --> Massive PE in a hemodylanmically unstable patient

A 4-year-old girl with sickle cell anemia is brought to the hospital with acute abdominal pain and extreme fatigue. She is afebrile. Her exam reveals tachycardia, hypotension, systolic murmur on heart exam, clear lungs, and fullness and tenderness in the left upper quadrant. Labs reveal a white blood cell count of 14800/microL, hemoglobin of 3.6 g/dL, and a platelet count of 180000/microL. Which of the following is the best initial step in the management of this patient? A. Broad-spectrum antibiotics B. PRBC transfusion C. IVIG infusion D. CT scan of the abdomen and pelvis

B. PRBC transfusion --> Splenic sequestion syndrome Sudden oncet pallor and signs of severe anemia

YOF presents for evaluation of her rash. She states while working in her shed yesterday she thinks she may have burned herself on a piece of hot pipe. She's not sure, but she was working in that area and felt pain. The pain is now significantly worse, and it feels like her whole arm is cramping. I don't know what is going on. It hurts so bad, I just don't feel good. I have a headache and just feel nauseous. Maybe I was in the heat too long.

Black widow spider bite

How do you appropriately size a cuff?

Bladder width >40 percent arm circumference Bladder length >80 percent arm circumference

A 65-year-old man arrives at the emergency department for altered mental status. He has a past medical history of hypertension, obesity, and metabolic syndrome. His blood pressure is 86/58 mm Hg, heart rate 120/min, respirations are 14/min, and spo2 is 99% on room air. He appears disheveled, the sclera is icteric, disoriented, and becomes agitated when the clinician palpates the right upper quadrant of his abdomen. After initial management with intravenous fluids and broad-spectrum antibiotics, the patient undergoes an MRCP after a gallstone was noted on an abdominal sonogram. The MRCP revealed a stone at the neck of the cystic duct and dilation of cystic duct and gallbladder. Several hours after the procedure, the patient becomes hemodynamically unstable. What is the most appropriate next step in management? A. Laparoscopic cholecystectomy B. Percutaneous cholecystectomy C. Exploratory laparotomy D. Ursodeoxycholic acid

B. Percutaneous cholecystectomy --> Cholangitis Fever, RUQ pain, jaundice Sepsis --> Unsuitable for surgery

Which of the following is a systemic manifestation of infective endocarditis? A. Hemarthosis B. Petechiae C. Cafe au lait spots D. Bronzing of the skin

B. Petechiae → Most frequent manifestation of vasculitis, seen on palate or palpebral conjunctiva Hemarthrosis → Bleeding into joint cavity, hemophilia, anticoags or antiplatelets Cafe au lait spots → Birthmark Bronzing of the skin → Addison's, hemochromatosis

What is an early sign of tension pneumothorax? A. Tracheal deviation B. Respiratory distress C. Increased cardiac output D. Epistaxis

B. Respiratory distress JVD and tracheal deviation are late signs

A 36-year-old female with migraines and anxiety presents to the emergency room with complaints of headaches, fever, and diarrhea. She is agitated, diaphoretic, and tachycardic, but normotensive. On neuro exam, she has hyper-reflexia without spasticity. Her CT head and MR brain are normal. A lumbar puncture is performed which reveals normal cerebrospinal fluid studies. Which of the following is most likely responsible for this patient's condition? A. Substance abuse B. Serotonin syndrome C. Neuroleptic malignant syndrome

B. Serotonin syndrome

A 35-year-old female is brought to the emergency department with complaints of palpitations and dizziness. She states the symptoms started after she went out jogging and have been present for the past 15 minutes. On examination, she is diaphoretic and tachypneic. Vitals: BP 80/50 mmHg, pulse 145 bpm, respiratory rate of 25 rpm, and a SpO2 of 85% on room air. She is started on supplemental oxygen and intravenous saline. The monitor shows regular, broad complex tachycardia. What is the most appropriate next step in her management? A. Loading dose amiodarone 150 mg over 10 minutes B. Synchronized cardioversion at 100 Joules C. Unsynchronized cardioversion at 200 Joules D. Intravenous bolus 6 mg adenosine

B. Synchronized cardioversion at 100 Joules --> Vtach Broad complex reg tachycardia, palpations, dizziness All patients with tachycardia who are hemodynamically stable should be cardioverted

Which of the following physical exam findings would you expect to find and why? BMP BG 102 Na 134 BUN 14 Cr 0.9 K 4.4 CO2 14 Cl 96 A. Tachycardia B. Tachypnea C. HOTN D. Fever

B. Tachypnea Low CO2 --> Must be hyperventilating and blowing it off

Which of the following statements regarding the PECARN guideline for children with head injury is INCORRECT? A. A child less than 2 years old with altered mental status definitely needs a CT scan. B. The PECARN guideline applies to children with GCS of 13 to 15. C. A child greater than 2 years old without any neurologic deficits and no physical signs of trauma has only a <0.05% risk of having traumatic brain injury. D. A child greater than 2 years old with a GCS of 15 but complains of severe headache may be placed under observation if a CT scan is unavailable. E. A child less than 2 years old with a GCS of 14 has a 4% risk of having traumatic brain injury.

B. The PECARN guideline applies to children with GCS of 13 to 15.

An examination revealing a high riding prostate and blood at the meatus are indications of which of the following? A. Kidney trauma B. Urethral injury C. Bladder injury D. Rectal injury E. Colon injury

B. Urethral injury

Hypotension leads to decreased renal perfusion. Decreased renal perfusion impairs urinary excretion of waste products, causing elevated levels of blood urea nitrogen and creatinine. Which lab will you order to look for this? How does uremia present?

BMP, CMP, renal function Confusion

A patient who has systemic hypothermia will develop respiratory depression (rr 8). If you ordered a BMP and ABG how would the following be affected? BMP CO2 - (high or low and why) Remember this is bicarb ABG pH - (high or low and why) PaCO2 - (high or low and why) This is carbon dioxide Does the patient have respiratory acidosis, respiratory alkalosis, metabolic acidosis, or metabolic alkalosis? How do you know.

BMP: CO2 is low because bicarb is trying to compensate ABG: pH= Low because acidosis, PaCO2= High because retaining Respiratory acidosis from respiratory depression

A patient arrives via EMS with report of an intentional overdose on Xanax. Which medication should you give?

Benzo OD --> Flumazenil

Which spider bite can also be associated with systemic symptoms?

Black widow

A 68-year-old female presents to the ED with abdominal pain, bloating, and nausea which she states began this morning upon wakening about 3 hours prior to arrival. She appears moderately uncomfortable and pale. She is urgently brought to an acute care room and evaluated by the resident physician. Her initial vital signs reveal tachycardia of 120 beats/minute, blood pressure of 103/52, oxygen saturation of 90% room air, respiratory rate of 24 breaths/minute, and an oral temperature of 99.0° F. Prior medical history includes hypertension and hypercholesterolemia, and she has a surgical history of two caesarean sections, bilateral tubal ligation, and open cholecystectomy. She says it is worse "right in the middle," and she "feels like vomiting but hasn't yet". She does report belching several times with a "foul smell". Her daughter and granddaughter are sick with a stomach virus, and she thinks she has the same. Her last bowel movement was the evening prior and was formed. She is rolling on the bed. She is tender over her upper quadrants diffusely with guarding but no rebound. Her exam otherwise is unremarkable. Analgesic and anti-emetic medicine is administered, as well as a bolus of intravenous fluid. Labs results reveal a leukocytosis of 15, lactate of 5.3, and normal lipase and CMP. Urinalysis is unremarkable. Coffee bean sign.

Bowel obstruction

Envenomation from which spider can result in necrotic ulcerations which require surgical consultation for debridement. Black widow or Brown Recluse?

Brown recluse

What is the Battle sign and what does it represent?

Bruising behind the ear Sign of basilar skull fracture

What is a seatbelt sign and what does it represent?

Bruising in the line of a seat belt Intraabominal injury

What is raccoon sign and what does it represent?

Bruising under the eyes Basillar skull fracture

Pruritic rash, patient on Lasix, negative nikolsky

Bullous Pemphigoid

A 28YOM presents to ED via EMS after being involved in a roll-over accident on the highway with report of lower back pain. A fracture is noted on CT. What is the #1 do not miss diagnosis? What symptoms will you ask about and document the presence or absence of on every single case. What physical exam will you perform (min 3) in which you will also document the presence or absence of?

Cauda equina syndrome Urinary/fecal incontinence /retention, saddle anesthesia, foot drop, priapism

An adult patient weighing 100 kg has boiling water spilled on his anterior chest and abdomen and complete left arm, which led to a wound that is painful, pinkish in color, and has slightly decreased sensations. Using the modified Parkland formula, how much total intravenous fluid should he receive per hour after 8 hours during the first day? A. 675 mL B. 5400 mL C. 337.5 mL D. 10,800 mL

C. 337.5 mL 4 mL of lactated Ringer's (LR) x the patient's weight in kg x percent total body surface area (TBSA) burned = total fluid in first 24 hours Only partial-thickness (second degree) and full-thickness (third-degree) burns are included in the total body surface percent calculation. Half of the fluid is given in the first 8 hours and the other half over the next 16 hours.

A 67-year-old man with a history of chronic obstructive pulmonary disease (COPD) becomes dizzy while working in his garden. His wife calls emergency medical services (EMS), and he is barely conscious when they arrive. His blood pressure is 75/40 mmHg, and he is pale. He is breathing, and his lungs are clear. ECG shows sinus tachycardia but is otherwise within normal limits, and he is free of chest pain. Diffuse urticaria is present, his lips and face are swollen with no signs of stridor, and he has a large local reaction to what appears to be an insect sting. Which of the following should be done with regard to the management of this patient? A. Intubate the patient and administer epinephrine B. Administer diphenhydramine and hydrocortisone, and avoid epinephrine due to the risk of precipitating cardiac arrhythmias C. Administer epinephrine intramuscularly, and establish an intravenous access D. Administer hydrocortisone, and establish an intravenous access

C. Administer epinephrine intramuscularly, and establish an intravenous access

A mother brings her 16-year-old son, who is known to have sickle cell disease, to the emergency room with "difficulty breathing." He was well until approximately 1 week ago when he began having bilateral leg and arm pain, which are his usual sites for sickle cell pain. During the past evening, the pain worsened, and he also began having fevers to 101.6 F (38.6 C). The patient has had several past admissions for his sickle cell disease for vaso-occlusive crisis pain management. He has never received a blood transfusion. On physical exam, the patient is awake and alert, but clearly in pain. His sclerae are icteric bilaterally. He has a 3/6 systolic ejection murmur, and he has decreased breath sounds in his right lung base. His blood count reveals a white blood cell count of 16,500, with a hemoglobin of 6.5 g/dL. and a platelet count of 426,004. His differential has 65% neutrophils, 25% lymphocytes, 8% monocytes, and 2% eosinophils, and his reticulocyte count is 18.6%. A chest radiograph reveals a small fluffy infiltrate in the right lower lobe. Which of the following should NOT be included in the management of this patient? A. Opioids for pain control B. Broad spectrum antibiotics C. Aerosolized nitrous oxide D. Incentive spirometry

C. Aerosolized nitrous oxide Acute chest syndrome Opiods --> Pain control to prevent "splinting", avoid respiratory deperession IV hydration --> Dehydration causes farther intravascular sickling, avoid over hydration --> Pulm edema Abx --> Cover possible infectious etiology Incetive spirometry --> Maintain alveolar potency

A 40-year-old female is brought to the emergency department due to altered mental status. She is accompanied by her sister, who states there is no history of recent illnesses or travel. Her sister recalls that she was seeing an endocrinologist a year ago but lost to follow up due to insurance issues. Vital signs are pulse rate 122/min, blood pressure 102/65 mmHg, respiratory rate 22/min, and temperature 101 F. Physical examination reveals •irritability •fine tremors •exaggerated deep tendon reflexes There is a rapid heart rate on auscultation. Which of the following complications can develop with this condition? A. Myocarditis B. Valvular heart disease C. Arrhythmia D. Pericardial effusion

C. Arrhythmia Thyroid storm

Which of the following is considered the antibiotic of choice in the treatment of human bite wounds? Why? A. Ampicillin B. Penicillin C. Augmentin D. Ciprofloxacin

C. Augmentin: Enhanced coverage of B Lactamase producing M. catarrhalis, H.flu, E.coli, B. fragilis and anaerobes, polymicrobial flora in the mouth Ampicillin → Will work bu IV so not first line Penicillin → Don't give because it does not cover B lactamase producing anaerobes and in bite words you have both Ciprofloxacin → No anaerobic coverage

A 30-year-old army reservist presented to a field clinic for localized numbness in bilateral feet that he has noticed this morning. He had been outside in cold rainy conditions for the past 2 days and states he was so tired from drill last night that he just went to bed without taking his shoes off. His blood pressure is 130/80 mmHg, pulse rate is 84 beats/min, and the temperature is 37 C (98.6 F). His examination of feet shows diffuse tissue maceration with decreased sensation. Dorsalis pedis pulse is palpable bilaterally, and strength is 5/5 in both feet. Which of the following could have best prevented the patient's condition? A. Application of topical terbinafine B. Appropriate management of blood glucose levels C. Drying his feet before going to bed D. Wearing warmer shoes

C. Drying his feet before going to bed --> Trench foot

A 68-year-old male with history of COPD is brought to the emergency department following a motor vehicle collision. On physical examination there is evidence of head trauma. The left side of the chest wall appears to move inward with inspiration and outward with expiration. A chest x-ray reveals multiple rib fractures on the left. Which of the following is the most appropriate intervention? What does this scenario represent? Best test to diagnose? A. Surgical fixation of the fractured ribs B. Application of elastic binders and adhesive tape C. Endotracheal intubation and mechanical ventilation D. Chest physiotherapy that encourage frequent coughing

C. Endotracheal intubation and mechanical ventilation: >65, head injury, flail chest, underlying lung disease, CXR Surgical fixation of the fractured ribs → If they have to open the chest for another reason fine but won't open chest just for that Application of elastic binders and adhesive tape → Stabilize chest, prevents chest expansion and makes it harder, if you dont practice breathing and exercise lungs → Atelectasis

A 25-year-old woman gravida 2, para 1 with a history of prior cesarean delivery is undergoing induction of labor. Suddenly the patient screams out in pain, becomes anxious, and starts to vomit. The patient has a heart rate of 140/min and a blood pressure of 82/42 mmHg. She has vaginal bleeding and hematuria. The fetal heart monitor shows a two-minute deceleration and a current fetal heart rate of 50/min. The tocodynamometer shows that the contraction intensity has greatly diminished. What is the best method to diagnose this patient's condition? A. Abdominal ultrasound B. Magnetic resonance imaging C. Exploratory laparotomy D. Hemoglobin and tests of coagulation

C. Exploratory laparotomy

Which of the following is a characteristic of a third-degree frostbite injury? A. Clear blister formation with surrounding erythema B. Nonsensate, central, white plaque surrounded by a ring of hyperemia C. Hemorrhagic blister formation D. Mottled tissue, with nonblanching cyanotic skin that eventually becomes dry, black, and mummified

C. Hemorrhagic blister formation

A 16-year-old girl is brought to the hospital with complaints of facial rigidity, muscle spasms, a painful abdomen, and headaches for one week. Recently, he has started feeling palpitations, loss of bladder control, and fecal incontinence. On examination, his blood pressure and heart rate fluctuate thrice in thirty minutes. She has a bandage on his right heel, which her mother says is from a piercing wound she got from a rusty nail. Her ECG reveals supraventricular tachycardia. She is given benzodiazepines for rigidity. Which of the following additional therapies is most likely to improve the patient's symptoms? A. IV propofol B. IV dantrolene C. IV magnesium D. IV calcium glucontate

C. IV magnesium --> Prevent muscle spasms

Which of the following should you administer to treat a patient with a temperature of 105F who is confused after being exposed to extreme temperatures in her home without air-conditioning? A. Acetaminophen B. Ibuprofen C. Ice water bath D. Lukewarm water bath

C. Ice water bath

A 16-month old child is brought to the emergency department immobilized in cervical spine precautions. The child was an unrestrained passenger in a moderate-speed rapid-deceleration MVC. You are concerned about possible Neck injury, keeping in mind that: A. Pseudosubluxation of C3 and C4 is common in children B. The small neck muscles make fracture more common than ligamentous injury C. If the child was ambulatory at the scene, spinal precaution are unnecessary

C. If the child was ambulatory at the scene, spinal precaution are unnecessary Psuedolucation happen C2-C3 Child has undeveloped neck muscles, larger/ heavier head --> Few fractured and more ligamentous injuries

A 12-month-old in the emergency department is diagnosed with possible viral meningitis. Which of the following cerebral spinal fluid (CSF) laboratory results is most consistent with this diagnosis? A. Decreased CSF glucose level and increased protein B. Decreased CSF total protein level and very few neutrophils C. Increased CSF mononuclear cells and normal glucose D. Increased CSF C-reactive protein (Indicates inflammation) and normal glucose

C. Increased CSF mononuclear cells and normal glucose Decreased CSF glucose level and increased protein= Bacterial

A 20-year-old man is brought to the emergency department by his friend after suffering a slash to the right side of the neck by a knife in a physical assault. On arrival, he is alert and awake but wincing in pain and covering his wound with his hand. His vital signs show a blood pressure of 109/75 mmHg and pulse 98/min. On examination, there is a 3 x 2 cm wound over the mid-portion of the neck. Careful palpation of the area reveals a large hematoma and brisk bleeding with the slightest pressure. Chest auscultation reveals mild stridor heard throughout the lungs. What is the best initial step in the management of this patient? A. CT angiogram neck B. Close the wound in the ED C. Intubation D. Neck x-ray

C. Intubation Hard signs (Expanding hematoma, active bleeding, shock, thrill, bruits) --> Intubation

Which of the following statements regarding the Glasgow Coma Scale is INCORRECT? A. It is mainly used to quantify level of consciousness in head trauma patients. B. The scoring may be influenced by non-trauma variables. C. It is useful in both trauma and non-trauma setting. D. A GCS score of 9 - 12 indicated moderate injury. E. One of its limitations is inter-observer reliability.

C. It is useful in both trauma and non-trauma setting.

You have a patient with a hypertensive emergency, currently having a STEMI. You are in a rural area and waiting on the cath lab team to come in for his procedure, but he has loss consciousness and you need to intubate. Which induction agent should not be used and why? A. Propofol B. Etomidate C. Ketamine D. Succinylcholine

C. Ketamine --> Sympathic stimulation, inc heart rate, contractility, MAP and cerebral blood flow

A 60-year-old female was found in a semi-comatose condition and brought to the emergency department. On exam her vitals are as follows. blood pressure was 80/40 mm pulse rate was 50 beats/min temperature 89.6F She is disoriented and lethargic. There is no palpable thyroid enlargement, but there is a scar noted to her anterior neck. Her heart sounds are distant, and her lungs are clear to auscultation. Laboratory test results demonstrate: hemoglobin 9 g/dl; hematocrit 25%; WBC count 9840 cells/mm3; platelet count 297,000 cells/mm3 creatinine 1.09 mg/dl sodium 126 meq/l; potassium 3.8 mEq/L; chloride 93 mEq/L; bicarbonate 24.5 mEq/L. Which of the following is most likely to be seen in the ECG of this patient? Explain A. ST depression and short QT interval B. Peaked T wave and short QT interval C. Low voltage ECG and long QT interval D. Low voltage ECG and peaked T wave

C. Low voltage ECG and long QT interval Distant heart sounds= cardiac tamponade

A 65-year-old man with sudden onset chest pain was admitted to the emergency department. His vital signs were a blood pressure of 95/60 mmHg, a pulse of 100/min, a respiratory rate of 19/min, and a normal temperature. His electrocardiography did not show any arrhythmia or block. His chest x-ray was remarkable for a widened mediastinum, while intravenous contrast CT-scan revealed double-lumen aorta. He was scheduled for emergent transesophageal echocardiography, which was significant for an intimal flap. What is the most appropriate next step in the management of this patient? A. Intravenous heparin and labetalol B. ICU admission and close monitoring C. Median sternotomy and hypothermic arrest D. Catheter guided thrombectomy

C. Median sternotomy and hypothermic arrest Flap --> Dissection --> Surgery

What is the diagnosis for a negative CT scan result in a child who underwent a CT scan according to the PECRAN guidelines? A. Normal brain findings B. Severe TBI C. Mild TBI D. Contusion E. Moderate TBI

C. Mild TBI

What is the immediate management of tension pneumothorax? A. Chest tube placement in 7th intercostal space B. Place a 3-sided dressing over chest tube site C. Needle thoracentesis in 2nd intercostal space D. ED thoracotomy

C. Needle thoracentesis in the 2nd intercostal space

Which of the following findings is consistent with thyrotoxicosis? A. Bradycardia B. Menorrhagia C. Nervousness D. Constipation

C. Nervousness

Patient 3 days post total hip replacement presents with inspiratory chest pain, fever, SOB, cool clammy skin and 80/50. What is the most likely cause of shock? A. Hypovolemic due to blood loss B. Neurogenic due to damage spinal cord C. Obstructive due to PE D. Cardiogenic due to heart failure

C. Obstructive due to PE

While exploring bat caves in South Africa, the patient was bitten in the neck. He was prescribed antibiotics, and the wound was cleaned before he returned to the United States. It has been three weeks since then, and he reports feeling ill with muscle fasciculations and seizure-like activity. He suddenly dies from cardiorespiratory arrest. The autopsy report reports Negri bodies in the hippocampus. Which of the following is a pathognomonic feature of this disease that the patient may have complained about initially? A. Continuous throbbing headache with emesis B. Flu-like symptoms that are unresponsive to over the counter medications C. Pain and/or itching at the inoculation site D. Fear of water or having air blow in the face

C. Pain and/or itching at the inoculation site

22 year old female patient comes in with delirium, urinary retention, flushed & dry skin. Her past medical history is significant for severe seasonal allergies (this time of year, conveniently), for which she takes Benadryl. You notice her eyes are also very dilated and she has decreased bowel sounds. Based on your suspected toxidrome, which of the following EKG findings would be most likely? A. ST elevation B. Peaked T waves C. QRS widening D. Sinus bradycardia

C. QRS widening

A 17-year-old male patient was referred to the emergency department following a motor vehicle collision. His vital signs on arrival to the emergency department were •blood pressure: 65/40 mmHg •pulse rate: 130 beats per minute •respiratory rate: 19/min •normal temperature He was absolutely conscious and complaining of severe abdominal and pelvic pain. Bilateral lung auscultation was clear. Exam revealed severe unstable pelvic fracture. He received 2000 cc Ringer's lactate, and his vital signs following resuscitation were blood pressure: 67/42 mmHg, pulse rate: 127 bpm, respiratory rate: 20/min, and normal temperature. What is the next preferred management? A. Emergent transfer to the operation room B. Transfusion 2 units packed cell C. Request FAST exam D. Request an abdominal CT-angiography

C. Request FAST exam

A 38-year-old patient presents to the emergency department after developing redness, irritations, and other signs of inflammation at the site of a sting by some insect while he was sleeping outside under a tree. In a similar incident that happened three years back, he has suffered an urticarial reaction because of sting and was treated with antihistamines and adrenalin. On physical examination, the site is erythematous and edematous. On further investigation, what of the following findings are most likely to be present? A. Secondary infection B. Serum sickness C. Retained stinger D. Delayed hypersensitivity reaction

C. Retained stinger

What is the appropriate treatment for frostbite? A. Slow warming B. Rapid warming C. Rubbing with warm hands D. Use of blanket

C. Rubbing with warm hands

A 5-year-old girl is brought to the emergency room after she was found chewing on an electrical wire at home. A burn is noted on the left oral commissure and adjacent lips. The child's parents deny any loss of consciousness or witnessed seizure. On physical examination, no additional burns or signs of trauma are noted on the child's body. Which of the following is the most likely complication necessitating close follow-up for this child, following discharge from the hospital? A. Ventricular fibrillation B. Respiratory arrest C. Severe hemorrhage D. Compartment syndrome Intraoral arc burn injuries can result in significant ______ and __________.

C. Severe hemorrhage Bleeding and airway compromise

A 24 YOF with a past medical history of Grave's disease is brought to the emergency room by her roommate. She is obtunded, heart rate is 128 per minute, the temperature is 104 degrees Fahrenheit, and the respiratory rate is 34 per minute. She is dehydrated and has scleral icterus. The patient's roommate mentions that the patient was vomiting for the last two days. A pregnancy test is positive. Thyroid stimulating hormone is 0.01, and free T3 is 20.0. Blood work shows mild leukocytosis without an obvious focus of infection. Alongside IV fluids and empiric antibiotics, how would you treat this patient? A. Start Lugol's iodine followed by Propylthiouracil and Propranolol an hour later; add hydrocortisone B. Start Lugol's iodine, add propylthiouracil and hydrocortisone an hour later; give radioiodine treatment two weeks later C. Start propranolol and propylthiouracil followed by Lugol's iodine an hour later; add hydrocortisone D. Start propranolol, propylthiouracil, and hydrocortisone; add Lugol's iodine an hour later; then give Radioiodine therapy two weeks later

C. Start propranolol and propylthiouracil followed by Lugol's iodine an hour later; add hydrocortisone --> Thyroid storm Radioioinine is CI in PG Non-PG patient --> Radioiodine treatment can be considered after 10-14 days of treating with thionamide, b-blockade and Lugol's iodine.

66YOM presents to ED found unconscious in a burning home, CPR was started in the field. He has a massive amount of burns and you need to intubate. When choosing your induction agent. Which induction agent should you not choose? Why? A. Ketamine B. Propofol C. Succinylcholine D. Etomidate

C. Succinylcholine Patients with burns, nerve damage or neuromuscular disease, closed head injury, dialysis patient, and other trauma may develop proliferation of extrajunctional acetylcholine receptors If the proliferation of extrajunctional receptors is great enough, sufficient potassium may be released to result in cardiac arrest

What is the recommended airway management for trauma patients with severe facial and neck injuries? A. Nasotracheal intubation B. Orotrahceal intubation C. Surgical cricothyrotomy D. Laryngeal mask airway E. Needle cricothyrotomy

C. Surgical cricothyrotomy

Which of the following is the most likely presentation of an acute pulmonary embolism in a patient without preexisting cardiac or pulmonary disease? A. Anginal chest pain B. Cough C. Tachypnea D. Palpatations

C. Tachypnea

Injury to what level of the cervical spine can result in diaphragmatic paralysis?

C3, C4, C5

Patient presents with dyspnea with exertion which has progressively worsened. Easily fatigued. Difficulty breathing while laying supine, waking up because they feel like they can't breathe. Bilateral lower extremity edema. Who's at risk?

CHF, make sure an MI didn't cause it Uncontrolled HBP, pregnancy, dilated CM, use cocaine, chemo, lupus, amyloidosis

When is the use of clinical decision rules like Canadian C-Spine and NEXUS contraindicated? What are these rules used for?

CI: <2, high risk injuries, direct blow to neck or penetrating trauma, comorbid conditions

If your patient has evidence of a globe rupture this may lead to compression of what nerve(s)? How will this present?

CN II --> Dec visual acuity, blurred vision, diplopia, inc blind spot

What are the 4 end-organs? What are signs and symptoms that they are not being perfused adequately?

CNS: HTN encephalopathy, stroke Renal: Acute renal failure Cardiopulmonary: CHF, ACS, MI, pull edema with respiratory failure, dissecting AA Ophthalmologic: Exudates, papilledema, retinal hemorrhage

What is the gold standard of diagnostic imaging when you suspect a vascular injury?

CT angiogram

When evaluating a hand injury, how will it present if there is an injury to the anterior interosseous?

Can't make an "Ok" sign, impaired movement of index/ middle finger, impaired flexion of thumb, index and middle finger, weakness in the middle and index finger

When should you order a head CT?

Canadian CT head rule

During the winter, a patient presents with headache, nausea, vomiting, and dizziness. A medical history reveals that the patient is homebound and conserving money by using his fireplace to keep warm. What is this patient at risk for and what is the mechanism that causes the problem?

Carbon monoxide posioning Binds to hemoglobin so O2 can't bind

Diffuse decreased condition on an EKG

Cardiac tamponade

STEMI, hypotension

Cardiogenic shock

A 51-year-old woman presents to the emergency department complaining of right upper quadrant, colicky abdominal pain onset two days ago. The patient states the pain has become constant for the past 8 hours, urging her to seek medical attention. The patient also complains of nausea and vomiting. During the physical exam, palpation to the right upper quadrant elicits pain. The patient's white blood count is 18,000 mm3, alkaline phosphatase (ALP) 55 IU/L, alanine aminotransferase (ALT) 30 IU/L, aspartate aminotransferase (AST) 40 IU/L, total bilirubin 1mg/dL, and lipase is 105 IU/L. What is the most likely diagnosis? Explain A. Choledocholithiasis B. Hepatitis A C. Acute pancreatitis D. Acute cholecystitis

D. Acute cholecystitis: + Murphys, RUQ pain, N/V

A 72-year-old man presents with acute left lower quadrant abdominal pain, nausea, non-bloody, non-bilious vomiting and constipation. On exam he is febrile (101° F) and his LLQ is tender with guarding and rebound. His white blood cell count is elevated. He has no prior history of gastrointestinal disease. Which of the following is the most likely diagnosis? A. Inflammatory bowel disease B. Irritable bowel syndrome C. Viral gastroenteritis D. Acute diverticulitis

D. Acute diverticulitis

A 17-year-old male patient presents to the out-patient department with low-grade fevers, malaise, myalgias, and headache for the past six days. He has also been experiencing tingling and numbness in his right forearm for two weeks. Three weeks ago, the patient went on a cave-exploring trip where he recalls a bat bite. He had no symptoms at that time, and so he did not consult a doctor. He has no medical problems and takes no medications. He does not smoke but drinks alcohol on weekends and parties. Vital signs show a blood pressure of 115/75 mm Hg, a pulse of 71/min, a respiratory rate of 16/min, and a temperature of 99 F (37.2 C). Which of the following may be seen in the patient's disease course? A. Hemorrhagic shock B. Disseminated intravascular coagulation C. Pneumonia D. Aerophobia

D. Aerophobia

A 55-year-old male presents with complaint of sudden ripping chest pain that radiates into the abdomen. On examination the patient is found to have diminished peripheral pulses and a diastolic murmur (Aortic regurgitation) . EKG reveals left ventricular hypertrophy (Indicates uncontrolled HTN). Which of the following is the most likely diagnosis? A. Acute MI B. Pulmonary embolism C. Acute pericarditis D. Aortic dissection

D. Aortic dissection

A 65-year-old male presented in the emergency department with altered sensorium, irritable behavior, and fever. His symptoms developed few hours after he went out to buy groceries in the hot weather. His past medical history is unremarkable. His vitals show a temperature of 104F, a pulse rate of 120 beats per minute and blood pressure of 110/80 mmHg. On examination, his mucous membranes are dry and there is a decreased elasticity of his skin. Which lab abnormality will most likely be present in this patient? A. Elevated creatine phosphokinase (CPK) 2000 U/L B. Creatinine 1.5 mg/dl C. Troponin 0.9 ng/mL D. Aspartate aminotransferase (AST)1500 U/L and alanine aminotransferase (ALT)1200 U/L What do these labs represent?

D. Aspartate aminotransferase (AST)1500 U/L and alanine aminotransferase (ALT)1200 U/L: Liver is very susceptible to heat damage CPK= Muscle damage Creatine= Renal function damage Troponin= Cardiac damage AST/ALT= Liver damage

a 35 year old patient complains of RUQ pain that has been worsening over the last few hours. She is febrile and there is a yellow tint to her skin. Her labs show leukocytosis, increased alk phos with increased GGT, increased bilirubin and increased ALT and AST. What is the gold standard diagnostic exam? A. US B. CT C. MRI D. Cholangiography via ERCP

D. Cholangiography via ERCP

Which of the toxidrome is known for its copious secretions? A. Anticholinergic B. Sympathomimetic C. Sedative/ hypnotic D. Cholinergic

D. Cholinergic

A 52-year-old male with a history of major depressive disorder presents to the emergency department complaining of a 24-hour history of severe abdominal pain, nausea, and vomiting. His blood pressure is 140/80 mmHg, pulse 82/min, respirations 16/min, and temperature 101.0 F (38.3C). Abdominal examination reveals localized tenderness over the right iliac fossa, guarding, and a positive Rovsing sign. The patient is shifted for an urgent appendectomy. In addition to routine anesthetics, the patient receives fentanyl and ondansetron intraoperatively. Shortly after transfer to the recovery unit, the patient becomes diaphoretic, agitated, and confused. His pupils dilate, heart rate is 110/min, and blood pressure is 198/100 mmHg. His temperature is 102.0 F (38.9 C). What is being presented (clues)? What was the cause? What is the most appropriate treatment for this patient? Any particular reason why you cannot use the other medications? a. Olanzapine B. Butalbital C. Ergotamine D. Cyproheptadine

D. Cyproheptadine Serotonin syndrome due to SSRIs + Ondansent and fentanyl Inc body temperature, hyperrelfexia, tremor, diaphoresis, N/D, seizures

A 70-year-old male presents to the hospital with an altered mental status. Physical examination is remarkable for bradycardia, anasarca, and hypothermia. His family reveals that he had thyroid cancer and a thyroidectomy. He has not been seen by a healthcare provider in 5 years. Which of the following is the most accurate regarding this condition? Explain your answer A. Increased respiratory rate and decreased blood pressure B. Decreased respiratory rate and increased blood pressure C. Decreased serum potassium level and decreased serum sodium level D. Decreased serum sodium level and decreased blood pressure

D. Decreased serum sodium level and decreased blood pressure Myxdema crisis

What is the cause of respiratory impairment in cervical spine injuries at levels C3- C5? A. Tracheal collapse B. Laryngeal paralysis C. Injury to the mainstem bronchi D. Diaphragmatic paralysis E. Esophageal expansion

D. Diaphragmatic paralysis

A 74-year-old female presents with a four-day history of fever, chills, right upper quadrant abdominal pain, and jaundice. She has a history of cholelithiasis and chronic congestive heart failure requiring oxygen therapy. Her medications include digoxin, furosemide, and captopril. Blood work reveals elevated white blood cell count, alkaline phosphatase, and bilirubin levels. Besides antibiotics, what is the next step in her management? A. Obtain a hepatitis profile B. Biliary lithotripsy C. Surgery with exploration of the common bile duct D. Endoscopic retrograde cholangiopancreatography

D. Endoscopic retrograde cholangiopancreatography Cholangitis (Common bile duct) --> RUQ pain, fever, jaundice

A 16-year-old female presents in the emergency department with severe left leg pain. She describes the pain as deep and aching, and she can feel a burning sensation in her left lower leg. Her vitals on arrival show a blood pressure of 100/80 mmHg and a pulse rate of 110 beats per minute. On physical exam, firmness of the anterior left lower leg is noted on deep palpation. Also, there is decreased two-point discrimination. Pain is increased with a passive extension of her left knee. What is the definitive treatment for this patient? A. Analgesia B. Leg elevation C. Angioplasty D. Fasciotomy

D. Fasciotomy --> Compartment syndrome

Patient presents with a history of gallstones and today they have N/V and painless jaundice. Which of the following diagnostic findings would be consistent with the patients presentation? A. Biliary tract infection 2ry to obstruction B. Gallstone found in gallbladder C. Gallstone causing infection in cystic duct D. Gallstone in the common bile duct causing ductal dilation

D. Gallstone in the common bile duct causing ductal dilation

A stuporous patient is brought to the emergency room with a five day history of progressive lethargy and confusion along with polyuria and polydipsia. On examination the patient is dehydrated and is without Kussmaul respirations BG 1200 mg/dL (75-110 mg/dL) Serum pH is 7.5 Na+ 150 mEq/L (136-146 mEq/L) Serum osmolality is 320 mosm/kg (280-300 mosm/kg) Urinalysis +BG, otherwise WNL What is the most likely diagnosis? A. Primary hyperaldosteronism B. Diabetic ketoacidosis C. Lactic acidosis D. Hyperglycemic hyperosmolar state

D. Hyperglycemic hyperosmolar state No kussumal, pH is >7.3, no ketones

For a trauma patient presenting to the ED, which of the following is an indication to intubate? A. GCS score of 12 B. O2 saturation of 92% C. Talking clearly but without making any sense D. Inability to phonate E. Respiratory rate of 16 breaths per minute

D. Inability to phonate

A 29-year-old male presents with complaint of substernal chest pain for 12 hours. The patient states that the pain radiates to his shoulders and is relieved with sitting forward. The patient admits to recent upper respiratory symptoms. On examination vital signs are BP 126/68, HR 86, RR 20, temp 100.3 degrees F. There is no JVD noted. Heart exam reveals regular rate and rhythm with no S3 or S4. There is a friction rub noted. Lungs are clear to auscultation. EKG shows diffuse ST segment elevation. What is the treatment of choice in this patient? A. Pericardiocentesis B. Nitroglycerin C. Percutaneous coronary intervention D. Indomethacin

D. Indomethacin --> Pericarditis

An 85-year-old male with a history of diabetes mellitus and high blood pressure presents to the emergency department with hoarseness, left temporal headache a painful left ear. His blood pressure is 127/75 mmHg, heart rate 95 beats per minute and temperature is 37.1 C. On physical examination the physician finds a red, tender left ear with an intact tympanic membrane with granulation tissue and exposed bone in the external canal. CT scan reveals osteomyelitis of the left temporal bone. Which of the following is the most appropriate management? A. Intravenous ceftriaxone B. Intravenous acyclovir C. Intravenous metronidazole D. Intravenous ciprofloxacin

D. Intravenous ciprofloxacin --> Malignant otitis externa from pseudomonas

A 35-year-old man presents with an eight-hour history of blistering and peeling rash, fever, and malaise. The patient has a past medical history of hypertension, hyperlipidemia, and focal seizures. His vital signs show oxygen saturation 98% on room air, respiratory rate 18 per minute, heart rate 90 beats per minute, blood pressure 110/80 mmHg, and temperature 102 F. On examination, the patient has a desquamative, erythematous rash over the back, abdomen, and chest. Blisters are observed in the mouth, throat, and eyes. Which of the following medications is most likely responsible for the patient's symptoms? A. Atorvastatin B. Lisinopril C. Diazepam D. Lamotrigine

D. Lamotrigine --> SJS

A patient presents to the emergency department with right upper quadrant pain over eight hours, nausea, and vomiting. On exam there is a fever of 101.2 degrees F. Ultrasound shows a distended gallbladder. What is the most appropriate management of this patient? A. Oral analgesics B. Diagnostic peritoneal lavage C. Proton pump inhibitors D. Laparoscopic cholecystectomy When would the wrong answers be indicated?

D. Laparoscopic cholecystectom --> Cholecystits Diagnostic peritoneal lavage --> Abdominal injury PPI --> Peptic ulcer

A 50-year-old patient presents complaining of headache and left eye pain for 5 hours. The patient admits to decreased vision in the left eye. The patient also complains of associated nausea. Which of the following is most likely on physical examination? A. Neovascularization B. Central vision loss C. Impaired red reflex D. Moderately dilated, nonreactive pupil What is this scerniro describing?

D. Moderately dilated, nonreactive pupil --> Glaucoma Neovascularization → DM Central vision loss → Macular degeneration Impaired red reflex → Cataracts

Which of the following is NOT a part of the breathing assessment? A. Monitor for oxygen saturation. B. Count for respiratory rate. C.Listen for equal bilateral breath sounds. D. Percussion of the chest E. Look for gestalt respiratory effort.

D. Percussion of the chest

Patient complains of chest pain that gets better sitting and leaning forward. Physical exam reveals distant muffled heart sounds and an EKG reveals low voltage and electrical altercans. What is the diagnosis? A. Pericarditis B. Myocarditis C. Endocarditis D. Pericardial effusion

D. Pericardial effusion

A 23-year-old patient presents with two days of fatigue, headache, fever and pain around the area in which she was bitten by a stray baby raccoon in an unprovoked attack 10 days ago. She cleaned the small wound thoroughly. Which of the following is the most appropriate intervention in this patient? A. Do nothing and treat her symptoms as it is unlikely a baby raccoon has rabies B. Human diploid cell rabies vaccine, 5 injections given all at once now C. Rabies immunoglobulin only D. Rabies immunoglobulins and human diploid cell rabies vaccine given 5 times in a 1-month period

D. Rabies immunoglobulins and human diploid cell rabies vaccine given 5 times in a 1-month period

A 32-year-old male presents to the emergency department from a construction site with a moderate pain to the index finger and palm of his right hand. The patient otherwise denies any other complaints. Physical exam shows a 3 mm puncture wound to the tip of his right index finger. The active range of motion to the index finger is limited. Plain radiographs of his hand show radiopaque fluid within the volar aspect of his index finger and palm. Which of the following is the next best step in the management of this patient? A. Admission to the hospital for close observation B. Corticosteroids and IV antibiotics C. Injection of a neutralizing agent D. Surgical debridement with a dose of IV antibiotics in the emergency department

D. Surgical debridement with a dose of IV antibiotics in the emergency department --> High pressure injury Surgical debrdidement and lavage within 6 hrs

A 76-year-old male presents after returning from a Safari in Africa. Seven days ago, he experienced chest pressure lasting one hour that did not respond to three sublingual nitroglycerin tablets. There was no ability to have lab work or an EKG. The pain has not returned. If the patient had a non-STEMI myocardial infarction, which of the following studies will still be positive? A. Electrocardiogram B. Myoglobulin C. CK-MB index D. Troponin I

D. Troponin I (7-10 days)

A 42-year-old male is brought to the emergency department with a stab wound to his right lateral chest wall. On physical examination, the patient is stable with decreased breath sounds on the right with dullness to percussion. An upright chest x-ray reveals the presence of a moderate pleural effusion. Subsequent diagnostic thoracentesis contains bloody aspirate. Which of the following is the next most appropriate intervention? A. Thoracotomy B. Needle aspiration C. Close observation D.Tube thoracostomy

D. Tube thoracotomy For hemothorax

On a cold New Year's Day, paramedics are called to respond to an unresponsive, 48-year-old, alcoholic man. On arrival, bystanders are performing cardiopulmonary resuscitation after they were unable to palpate a pulse. The patient is immediately placed on the cardiac monitor, which revealed variable, wide QRS complexes. Paramedics resume cardiopulmonary resuscitation. Two rounds of defibrillation and 300 mg of amiodarone are given with minimal changes to the cardiac rhythm. Rectal temperature of 87.6°F (30.8°C) is measured. Which of the following is the next best step in the management of this patient? A. Give procainamide B. Give another round of amiodarone C.Defibrillate patient at 360J D. Warm the patient

D. Warm the patient --> Severe hypothermia

H/O uncontrolled DM and alcoholism. Presents to ED via EMS found unconscious at the park. Currently awake and reports abdominal pain, nausea, and vomiting x 2 days. BG 400, CO2 10, Anion Gap increased, EtOH 340, temp 102F, RR 22, O2 93% What is the most likely precipitating factor?

DKA Aspiration pneumonia from EtOH use

+ Sidel sign

Damage to cornea/ sclera

Define hypoexmia

Decrease in the partial pressure of O2 in the blood Pulse Ox <91% on room air ABG: PO2 <60 mmHG on room air or dec by 10 mmHG from baseline

What is the consequence of a blast crisis?

Decreased tissue perfusion End-organ damage Stroke Hyper viscous

Define hypoxia

Deficiency in the oxygenation at tissue or cellular level --> Anaerobic metabolism, cellular acidosis, cell death, organ failure

How would a patient with a partial airway obstruction present?

Diminished or noisy air entry

Early septic shock (warm or hyperdynamic) causes reduced diastolic blood pressure. Vasodilation results in more blood to the skin/ periphery and thus skin is flushed and warm. Capillary refills will be brisk. Preload is increased and thus cardiac output is increased initially as well. As sepsis continues/ becomes more severe, vasoconstriction occurs and thus preload is decreased, and cardiac output is decreased. What type of shock is septic shock? Why does early septic shock cause reduced diastolic pressure? Will the pulse pressure be narrow or wide? Explain.

Distributive shock Early shock causes both increased cardiac output and severe vasodilation The only shock to result in increased cardiac output and vasodilation -->

Inflamed abnormal pouching of the gastrointestinal lumen which developed within naturally weak places of the colon. Most commonly the sigmoid colon. How will it present? What complications can occur if you miss this diagnosis?

Diverticulitis LLQ pain, N/V/D, fever Perforation, abscess, fistula

Thumb sign on an x-ray. What are the symptoms?

Epiglottitis --> Dysphagia, drooling, distress

What is the estimated percentage risk of brain injury for a child below two years old with head trauma who does not show any neurologic deficits but was noted to have scalp hematoma? A. 4% B. 5% C. 2.5% D. <0.02% E. <1 %

E. <1 %

Which of the following are associated with pelvic fractures? A. Abdominal injury B. Rectal injury C. Kidney injury D. Colon injury E. Bladder and urethral injury

E. Bladder and urethral injury

A trauma patient presents in the trauma bay with hypotension, tachycardia, and altered mental status. After completing your ABCs, what is your next step? A. CT scan B. Antibiotics C. MRI D. Interventional radiology E. Intravenous fluid therapy

E. Intravenous fluid therapy

Which of the following statements regarding the primary survey is NOT true? A. It includes assessment of the airway, breathing, and circulation. B. One of its goals is to identify life threats. C. One of its goals is to provide immediate stabilization. D. It is the standardized initial assessment of all trauma patients. E. It involves arriving at the correct diagnosis at the time of the assessment.

E. It involves arriving at the correct diagnosis at the time of the assessment

According to the Canadian Criteria for head CT, a cranial scan is indicated in patients with head injury whose GCS scored remains below 15 after how many hours? A. One hour B. Half an hour C. Three hours D. Four hours E. Two hours

E. Two hours

20 weeks gestation or greater BP > 140/90 + proteinuria or BP > 140/90 + end-organ damage Platelets <100K Renal insufficiency Cr > 1.1 Impaired liver function: LFTs > 2x normal Pulmonary edema Cerebral or visual symptoms Patient is now having uncontrolled electrical disturbances in her brain resulting in change in behavior with impaired awareness (change or loss of consciousness or awareness). May have associated involuntary muscle movements or sensory symptoms such as tingling, dizziness, and flashing lights

Eclampsia

Is rhabdomyolysis more common in an injury sustained by lightening or from an electrical source?

Electrical

55 YOM with a history of diabetes and chronic non-healing wound who has been coming to the wound clinic for several months' reports development of a fever 2 days ago and note of chest pain today. He states his pain is constant and does not worsen or improve if he changes position. It does not get worse with exertion. On exam, he notably has a murmur which he states is new.

Endocarditis

Which presents on CT as a convex shaped bleed?

Epidural

Which type of bleed is arterial and thus affected by blood pressure and may expand rapidly?

Epidural

Patient presents with radicular back pain which is aggravated with movement and bending over. May have just progressively worsening back pain. May have any neurologic deficits. May have signs and symptoms of sepsis without prominent back pain Vitals: 100.4F, 140/90, 99bpm, 18rr, 98O2 What is radicular pain? What are you looking for on exam? Who's at risk?

Epidural abscess Radicular pain: Radiates from your back and hip into your legs through the spine Redness, tenderness to palpation IC, IV drugs, post-op

What is the difference between epidural and subdural bleeds?

Epidural: Damage to the arteries within the epidural space Subdural: Damage to the veins within the subdural space

Life threatening condition characterized by upper airway inflammation and obstruction Most commonly caused by beta hemolytic strep, staph aureus, Haemophilus influenzae S/sx: fever, sore throat, odynophagia accompanied by drooling Children often also have cough, stridor, tripoding, dyspnea, muffled phonation/ dysphonia, toxic appearance What physical exam maneuver should be avoided if you suspect this diagnosis?

Epiglotitis Use a tongue depressor is CI

Define acute respiratory failure

Inability to ventilate adequately or provide sufficiency O2 to the blood and systemic organs --> Hypoxemia and hypercapnia

Define shock

Inadequate tissue oxygenation

What are the causes of hypercapnia respiratory failure?

Increased CO2 production: Fever, sepsis, burns Decreased alveoli ventilation: Dec RR, dec tidal volume, inc dead space

ST elevation in II, III and aVF, what artery and how does it affect the rate?

Inferior MI --> RCA --> Bradycardia

If your patient has an inferior orbital wall fracture, which structure can potentially be injured? How will this present?

Inferior rectus muscle can become entrapped --> Dec visual acuity, loss of EO movements, eye can't look up

How will esophageal varacies present? Who is at risk? how is it treated? Why does this medicine work?

Inflamed veins in the esophagus → Asymptomatic → Rupture sx: coffee ground emesis, melena, caput medusa, ascites, icterus RF:Pts with portal HTN, cirrhosis MC cause in adults, alcohol abuse Degree of cirrhosis directly correlates with rupture Early cirrhosis= High LFTS, late= Low LFTS → More likely to bleed Treatment: BB (Stabilized or just found them), rubber band ligation, IV octreotide (Standard) Works: Decreases BP, octreotide is a vasoconstrictor somatostatin analog that dec portal blood flow, slows blood flow to engorged veins which stops the bleeding

What is ARDS? What are the diagnostic findings? How do you treat it? If you need to give IV fluids, would you give colloid solutions or crystalloid solutions and why?

Inflammation of the lungs → Fluid leakage into lungs → Dec blood O2 (V/Q mismatch) Type 1 pneumocytes damage → Not surfactant → Stiff, don't stretch, do not hyperinflate Causes: Infection, septic (MC RF), aspiration of gastric content (2nd MC RF), trauma, massive transfusion, OD, drowning Dyspnea, organ failure, severe refractory hypoxemia, CXR: Bilateral pulm infiltrates, PCWP< 18mmHG (Differentiate from CHF), 7 days of inciting event CPAP to force air through, treat underlying cause, LMWH for prophylaxis Crystalloid (Normal saline, saline + dextrose, lactated ringers): Give but not too much Colloids (Albumin, fresh frozen plasma): Help bring fluid out, risk for anaphylaxis, give then give lasix to urinate it out

70 YOM with a history of CAD presents to ED with report of abdominal pain, nausea, vomiting. States this pain has been intermittent over the last 3 days. Denies any changes in bowel movements or difficulty with urinating. Last meal was 2 days ago secondary to pain and nausea when he eats.

Intestinal ishemia 2ry to CAD

Describe the indication for the transfusion of blood products during the management of the patient with a traumatic injury resulting in compromised circulation?

Patients who partially respond to initial fluid resuscitation or do not respond at all Estimated blood loss >1500mL (>30% of blood volume)

Air under the diaphragm

Perforated bowel

Diffuse ST elevation, PR elevation in AVR

Pericarditis

32 YOM presents to ED with report of chest pain and shortness of breath. He states he's currently recovering from the flu (diagnosed in ED 2 days ago) and is concerned he may have pneumonia. He reports occasional dry cough. States pain is constant, but worse at night when he tries to sleep. Patient appears in pain and is diaphoretic. +JVD, lungs clear bilaterally. Vitals 90/50, pulse 120

Pericarditis --> Obstructive shock

19 YOM presents to ED with report of right-side throat pain x 3 days with fever which developed today. Admits to dysphagia and odynophagia and states it hurts to swallow so he is notably spitting out his saliva. On exam there is no dysphonia, 2+ tonsillar hypertrophy without exudates and uvula is midline.

Peritonsilar abscess

If your patient has a lateral knee injury, which nerve may be injured and how will this present?

Peroneal nerve Foot drop, decrease sensation to the 1st and 2nd toe

Loss of gastrointestinal wall integrity with subsequent leakage of enteric contents. Direct trauma or tissue ischemia and necrosis lead to full-thickness disruption of the gastrointestinal wall and perforation. When may this occur?

Perorated viscous Diverticulitis, PUD, IBD

Can't retract foreskin

Phimosis

If a patient has significant oral trauma with a maxilla fracture, what should you do?

Place a nasopharyngeal airway

If the tongue is in the way of the airway, what should you do?

Place oropharyngeal airway to hold it down

Abnormal collection of air in the pleural space between the lung and the chest wall. Give me the diagnosis and signs and symptoms.

Pneumothorax Chest pain, dyspnea, dec breath sounds, unequal expansion, hyperresonace to percussion

A 25-year-old female presents to the emergency department after an episode of substernal chest pain with radiation to the middle of her back that came on suddenly and lasted for about four minutes this morning while in bed. She denies previous episodes. Examination is unremarkable, but she appears jittery. Toxicology screen is positive for cocaine. Which of the following medications is contraindicated in this patient? Why? A. Lorazepam (Ativan) B. Dilitazem (Carbdizem) C. Nitroglycerin (Nitrostat) D. Propranolol (Inderal)

Propranolol (Inderal) → Can increase vasospasms because of unopposed alpha-1 constriction Alpha naturally cause vasoconstriction, Beta naturally cause vasodilation

Describe the role of evaluating lactic acid levels when evaluating patients with traumatic injury

Providence evidence of hypoxia

Ms. Smith is a 68YOF who underwent hip replacement surgery two weeks ago. Her postoperative period was complicated by RLL pneumonia, and the patient has been bed-ridden ever since. A nurse calls you to the patient's room due to vital sign abnormalities and complaints of chest pain. •HR 108 bpm •BP 90/60 •RR 35 rpm •O2 98% •Temp 100.2F •WBC count improved You note jugular venous distension and profound dyspnea. RLL with crackles, left lung clear, expansion is symmetrical Heart sounds are tachy, but regular, no murmur. No peripheral edema, no pulse deficits.

Pulmonary embolism Post ortho surgery, bed-ridden, chest pain, JVD, tachycardia, HOTN, febrile

What are some signs of blood loss >1500mL or >30% of blood volume

Pulse 120-140 Decreased SBP Pulse pressure decreased RR 30-40 Urine output 5-15ml/hr Anxious, confused

How do you monitor a response to fluid resuscitation?

Pulse, BP, MAP, pulse pressure, urinary output (0.5mL/kg/hr), capillary, lactic acid (Looking for 50% improvement)

What is an afferent pupillary defect? What does it present? What side is the lesion with right APD?

Pupils respond different to light, shining light into unaffected eye Constriction but shining light into affected eye Appear to dilate due to lack of normal constriction Have consensual not direct Severe retinal disease: CRVO, CRAO, retinal detachment, optic neuritis (MS), glaucoma, direct optic nerve damage with trauma, tumor behind eye Right side before optic chiasm

A mother brings a 3-month-old infant to the emergency department and states that the child has been vomiting for the past 2 weeks. The vomiting is often projectile, but it is not stained. It appears to be mostly food. The infant has no known illness. On examination, a small, olive-like hardness in the right upper quadrant is palpated, and many peristaltic waves are noted. What is most likely diagnosis and what is the imaging modality of choice? Fill in the blanks below to demonstrate the electrolyte abnormalities you would expect to see. Explain why you would expect to see these findings. BG: Na+: K+: BUN: Cr: Cl-: HCO3: pH: pCO2:

Pyloric stenosis, US BG: Normal Na+: Normal K+: Low from vomiting BUN: High Cr: Normal Cl-: Low HCO3: High (Metabolic alkalosis) pH: High pCO2: Norm- high

Low riding patella on x-ray

Quadriceps tendon rupture

What are some red flags of a temporal bone fracture?

Raccoon eyes, hemotympanum, Battle's sign, CSF leak, hearing problems or vertigo

Define hypercapnia

Retention of CO2

74-year-old man presents with sudden vision loss in his right eye. He has a medical history of hypertension and coronary artery disease and new onset atrial fibrillation. On physical exam, a carotid bruit is auscultated. A retinal exam is performed with note of a pale retina and red spot. What is the diagnosis? What are the clues? What is the most likely cause?

Retinal arterial occlusion Cherry red spot, unilateral vision loss, carotid bruit HTN, CAD, AFib

An 82-year-old male with a past medical history of hypertension, hyperlipidemia, and factor V Leiden mutation presents with a sudden loss of vision in the left eye. The patient has an absence of a relative afferent pupillary defect (RAPD). The fundus exam is significant for mild venous dilation and tortuosity with diffuse retinal dot/blot and flame-shaped hemorrhages. Which of the following other exam findings would be present with the underlying diagnosis? What is the most likely cause and etiology for this case?

Retinal vein occlusion Factor V --> Hypercoag, HTN, hyperlipidemia

What are the signs and symptoms of a renal injury?

Retroperineal --> Evidance of rib fractures, Grey-Turner's sign, hematuria, signs of shock is severe blood loss 3rd MC organ injured, MC in children, 90% due to blunt trauma

If a patient develops an uncal herniation, how may this present?

Rising ICP → Causes portions of the brain to flow from one intracranial compartment to another Sx of increased ICP: HA, N/V, AMS, Cushing's triad (Bradycardia, HOTN, irregular respirations/ apnea Acute loss of consciousness associated with ipsilateral pupillary dilation and contralateral hemiparesis → due to compression of CN III

What is the most common mechanism of subdural hematomas?

Rupture of bridging veins of the dural sinus due to ascending- descending type motion Elderly MC due to atrophy, ETOHs, abuse child

Which bullous rash will have a positive Nikolsky sign?

SJS Pemphigus vulgaris

Dec lung sounds, tracheal deviation, inc JVP, chest pain/ dyspnea

Tension pneumothorax

Acute onset unilateral pelvic or abdominal pain is the most typical presentation. Can be chronic, intermittent, positional, variable intensity, and may radiate to flank or groin. What physical exam findings are you looking for?Who's at risk?

Tesricular torsion: Cremasteric reflex, - phrens sign RF: Bell clapper defromity Ovarian torsion: Bimanual exam, pain or mass RF: Cysts, tumor, PG

The majority of people who die following a MVA succumb to what injury? A. Pelvic fracture and bleeding B. Thoracic injuries C. Spleen and liver injuries D. Brain injury

Thoracic injuries

If your patient has a posterior knee injury, which nerve may be injured and how will this present?

Tibial nerve: Loss of ankle plantar flexion, dec sensation to plantar aspect and arch of foot

22 YOM was punched in the eye and reports redness and severe photophobia with decreased visual acuity. On exam there is diffuse conjunctival injection with prominence of perilimbal; vessels (ciliary flush). The pupil is miotic and irregular. Minimally reactive with note of direct and consensual photophobia. You believe he has traumatic inflammation to which structure? If this structure swells with inflammation it may cause this ophthalmologic emergency ___________?

Traumatic iritis Glaucoma

A homeless patient has been walking around in the rain for days and presents with report of burning pain to his feet and you notice large blisters on the soles of both feet. What is his most likely diagnosis?

Trenchfoot

60 YOM with a history of DM2 presents to the emergency department with report of substernal chest pain which has been constant since onset 30 minutes ago while shoveling; "it's just not getting better." "It feels worse, it's just so tight! I couldn't take it so, I called 911." The patient was given nitroglycerin in the ambulance, but he only reports minimal improvement in the pain. An EKG was ordered and demonstrates a normal sinus rhythm with a pre-ventricular contraction in lead II and t wave inversion in V5 and V6. His troponin level was ordered at the time of presentation and was repeated 3 hours later. Both enzymes were within normal levels. Patient denies any similar episodes

Unstable angina

Pt has severe menorrhagia and reports gum bleeding when they brush teeth

VWB disease

Define responder, partial responder, non-responder

Vitals signs return to normal --> Rapid response/ responder Transient improvement in vitals, recurrence of dec BP, and inc HR --> Transient response Vitals remain abnormal --> Non-responder --> Will require immediate intervention, FAST exam, exploratory surgery

A 72-year-old male presents to the emergency department with a fever and swollen, painful right knee for one day. His past medical history is significant for diabetes mellitus and two gout attacks in his left foot. His temperature is 38.0 C (100.4 F), blood pressure 125/84 mmHg, and heart rate 95 beats per minute. Physical exam shows a swollen, warm and erythematous right knee with \ pain on passive and active motion. Labs show hemoglobin 13.0 g/dL and leukocyte count 14500/microL. What is the most appropriate next step in managing this patient? A. CT angiogram of knee B. Arthrocentesis C. Knee X-ray D. MRI of the knee

b. Arthrocentesis Septic arthritis

What are the normal values for an ABG?

pH: 7.35-7.45 CO2: 35-45 pO2: 80-100 HCO3: 22-26 O2 sat: 95-100%


Ensembles d'études connexes

Audit Chapter 12 Multiple Choice

View Set

UNIT III: Immunity Part II (Adaptive Immunity)

View Set

BUS 303 - International Business

View Set

Chapter 25 / 34 Abuse and Neglect

View Set

US History (Texas) Ch. 8 The Jefferson Era

View Set

Mastering micro ch 12 hw when the bacteria away..fungi play

View Set

Muscles and Action of the Forearm, Wrist and Fingers Test

View Set

AP Comparative Government Iran Vocabulary

View Set